Đến nội dung

Ban Biên Tập

Ban Biên Tập

Đăng ký: 01-01-2012
Offline Đăng nhập: 01-08-2013 - 22:35
***--

#617579 Ngày 29 tháng 02

Gửi bởi Ban Biên Tập trong 29-02-2016 - 00:22

Ngày 29 tháng 2 chỉ xảy ra vào những năm nhuận. Ngày 29 tháng 2 là ngày thứ 60 trong một năm nhuận của lịch Gregory. Theo định nghĩa, năm nào có ngày này là một năm nhuận. Nó chỉ xuất hiện mỗi bốn năm một lần như 1996, 2000, 2004, 2008, 2012 và 2016.

 

Liên quan đến vấn đề này, một câu hỏi khác được đặt ra: Tại sao tháng 2 lại có 28 ngày(năm nhuận là 29 ngày) trong khi các tháng khác trong năm đều có 30 hoặc 31 ngày?

 

Vì những năm 46 trước Công nguyên, thống soái La Mã Julius César khi định ra lịch dương, quy định ban đầu là mỗi năm có 12 tháng, tháng lẻ là tháng đủ, có 31 ngày; Tháng chẵn là tháng thiếu, có 30 ngày. Tháng 2 là tháng chẵn lẽ ra cũng phải có 30 ngày. Nhưng nếu tính như vậy thì một năm không phải có 365 ngày mà là 366 ngày. Do đó phải tìm cách bớt đi một ngày trong một năm.

 

Vậy thì bớt đi một ngày trong tháng nào?

 

Lúc đó, theo tập tục của La Mã, rất nhiều phạm nhân đã bị xử tử hình, đều bị chấp hành hình phạt vào tháng 2, cho nên mọi người cho rằng tháng đó là tháng không may mắn. Trong một năm đã phải bớt đi một ngày, vậy thì bớt đi một ngày trong tháng 2 , làm cho tháng không may mắn này bớt đi một ngày là tốt hơn. Do đó tháng 2 còn lại 29 ngày, đó chính là lịch Julius.

 

Sau này, khi Augustus kế tục Julius César lên làm Hoàng đế La Mã. Augustus đã phát hiện ra Julius César sinh vào tháng 7, theo lịch Julius thì tháng 7 là tháng đủ, có 31 ngày, Augustus sinh vào tháng 8, tháng 8 lại luôn là tháng thiếu, chỉ có 30 ngày. Để biểu thị sự tôn nghiêm như Julius César, Augustus đã quyết định sửa tháng 8 thành 31 ngày. Đồng thời cũng sửa lại các tháng khác của nửa năm sau. Tháng 9 và tháng 11 ban đầu là tháng đủ thì sửa thành tháng thiếu. Tháng 10 và tháng 12 ban đầu là tháng thiếu sửa thành tháng đủ. Như vậy lại nhiều thêm một ngày, làm thế nào đây? Lại lấy bớt đi một ngày trong tháng 2 không may mắn nữa, thế là tháng 2 chỉ còn 28 ngày.

 

Hơn 2.000 năm trở lại đây, sở dĩ mọi người vẫn tiếp tục dùng cái quy định không hợp lý này chỉ vì nó là một thói quen. Những người nghiên cứu lịch sử trên thế giới đã đưa ra rất nhiều phương án cải tiến cách làm lịch, họ muốn làm cho lịch được hợp lý hơn.

 

Trở lại Lịch Gregory. Như chúng ta biết, Lịch Gregory là một bộ lịch mới do Giáo hoàng Grêgôriô XIII đưa ra vào năm 1582. Lịch Gregory chia thành 12 tháng với 365 ngày, cứ 4 năm thì thêm một ngày vào cuối tháng 2 tạo thành năm nhuận. Vì vậy theo lịch Julius thì một năm có 365,25 ngày. Nhưng độ dài của năm mặt trời là 365,242216 ngày cho nên lịch Julius dài hơn khoảng 0,0078 ngày trong một năm, tức là khoảng 11 phút 14 giây.

 

Để bù vào sự khác biệt này thì cứ 400 năm ta sẽ bỏ bớt đi 3 ngày năm nhuận. Cho đến năm 1582, thì sự sai biệt đã lên đến 10 ngày. Giáo Hoàng Gregory XIII quyết định bỏ 10 ngày trong tháng 10 năm đó để cho lịch và mùa màng tương ứng trở lại. Sau ngày 4 tháng 10 năm 1582 là ngày 15 tháng 10. Và để tránh sai biệt, lịch lấy năm nhuận là năm có số thứ tự chia hết cho 4 (như năm 1964, 1980, 2004, ...) và các năm tận cùng bằng 00 phải chia hết cho 400 mới là năm nhuận (năm 2000 chia hết cho 4 và 400 nên là năm nhuận, những năm 1700 1800 và 1900 chia hết cho 4 nhưng không chia hết cho 400 nên không phải là năm nhuận, ...). Lịch đã sửa mang tên lịch Gregory và được áp dụng cho đến bây giờ.

 

Ngày 29 tháng 2 ngày nay ra sao?

 

Nói về ngày này thì trước tiên : Đứa trẻ nào chào đời vào đúng ngày 29 tháng 2 mà muốn mừng sinh nhật đúng ngày,thì phải đợi đến 4 năm nữa. Lý do tháng này năm nay có ngày nhuần, có đến 29 ngày chứ không phải 28 ngày như của mọi tháng Hai khác.

 

Tại một số quốc gia, để tiện việc, chính phủ sở tại có thể cho đứa trẻ sinh ra trong ngày 29 tháng Hai được kê trong giấy khai sinh là sinh ngày 28 tháng Hai hay sinh ngày 1 tháng Ba. (Theo tập tục của Pháp. Tuỳ cha mẹ chọn ngày sinh 28 -29- hoặc 1tháng 3).

 

Ngoài ra ngày 29 tháng 2 còn là Ngày phái đẹp tỏ tình. Cứ 4 năm mới có một ngày 29/2, ngày này ở châu Âu được coi là “Ngày phụ nữ tỏ tình” - tức là phái đẹp chủ động cầu hôn với giới mày râu.

 

Đây là phong tục lâu đời ở Anh, người Scottland đã thông qua đạo luật lấy ngày 29/2 là “Ngày quyền lợi phụ nữ”. Hồi đó, Nữ hoàng Margarit đã tuyên bố: Trong ngày này phụ nữ có thể cầu hôn với đàn ông và tiến hành trừng phạt những gã đàn ông thích thả dê nhưng lại chối bỏ trách nhiệm.

 

Từ thế kỷ XVII, phong tục này đã lan ra khắp châu Âu. Ngày 29/2/2004 đã có hơn 7.000 phụ nữ Anh chủ động cầu hôn, trong đó có cô MC một đài truyền hình đã cầu hôn bạn trai ngay trên sóng truyền hình và đã thành công.

 

Hiện nay, những vị mày râu nào từ chối lời cầu hôn của bạn gái trong ngày này sẽ phải nộp 1 Bảng tiền “thế chấp” hoặc phải tặng một tấm áo lụa cho người con gái bị tổn thương, nhưng tình huống này rất hiếm khi xảy ra.

 

Theo thống kê, trong ngày 29/2 lần trước có tới 92% đàn ông được ngỏ lời đã vui mừng chấp nhận tình cảm của bạn gái, 4% lúc đầu không đồng ý vì bất ngờ, nhưng sau khi suy nghĩ đã vui vẻ chấp thuận. Đối với nhiều phụ nữ Anh, 29/2 là một ngày có ý nghĩa quyết định trong cuộc đời.

 

Trong lịch sử đã có rất nhiều nhân vật nổi tiếng đã nên duyên nhờ chủ động cầu hôn trong ngày này. Tiêu biểu là nữ minh tinh điện ảnh người Hungary G. Garbo. Bà từng tuyên bố: Cả 9 người chồng trong cuộc đời đều do bà chủ động cầu hôn và bà giải thích “phụ nữ phải hướng dẫn ý thích của đàn ông”.

a




#598136 Thứ Sáu ngày 13 dưới góc nhìn toán học và văn hóa

Gửi bởi Ban Biên Tập trong 13-11-2015 - 16:54

BÀI TẬP

 

Hãy chứng minh:

 

1) Một năm bất kỳ có ít nhất một thứ sáu ngày 13 và nhiều nhất ba thứ sáu ngày 13.

 

2) Một năm có ba thứ sáu ngày 13 khi và chỉ khi ngày đầu năm là thứ năm (đối với năm không nhuận) hoặc chủ nhật (đối với năm nhuận).

 

3) Khoảng cách giữa hai ngày thứ sáu 13 gần nhất chỉ có thể là 27, 90, 181, 244, 272, 335 hoặc 426 ngày.




#598134 Thứ Sáu ngày 13 dưới góc nhìn toán học và văn hóa

Gửi bởi Ban Biên Tập trong 13-11-2015 - 16:53

Ở một số nước phương Tây, người ta cho rằng thứ sáu ngày 13 là ngày rủi ro. Tuy nhiên, báo Le Figaro (Pháp) số ra ngày 11-3-2009 cho biết số người mua lô tô tại Pháp vào thứ sáu ngày 13 cao gấp 3 lần so với những ngày khác.

 

 

Fri13.jpg

 

 

Vậy thứ sáu ngày 13 có đặc điểm gì về mặt toán học và văn hóa? Nó là ngày tốt hay xấu?
Bằng lý thuyết đồng dư, toán học chứng minh được một năm bất kỳ có ít nhất một thứ sáu ngày 13 và nhiều nhất ba thứ sáu ngày 13. Hơn nữa, một năm có ba thứ sáu ngày 13 khi và chỉ khi ngày đầu năm là thứ năm (đối với năm không nhuận) hoặc chủ nhật (đối với năm nhuận). Đó là trường hợp của năm 2009: có ba thứ sáu ngày 13 rơi vào tháng hai, tháng ba và tháng mười một. Sự kiện này đã xảy ra vào năm 1998 và sẽ lặp lại vào các năm 2015, 2026.

 

Năm 2010 và 2011 chỉ có một thứ sáu ngày 13 mỗi năm. Năm 2012 có ba thứ sáu ngày 13 rơi vào tháng giêng, tháng tư và tháng bảy. Bộ ba “giêng, tư, bảy” này ít gặp hơn so với bộ ba “hai, ba, mười một”. Năm 2013 có hai thứ sáu ngày 13 rơi vào tháng 9 và tháng 12. Tổng cộng có 21 thứ sáu ngày 13 từ 2009 - 2019.

 

Cũng bằng toán học, ta tính được khoảng cách giữa hai ngày thứ sáu 13 gần nhất chỉ có thể là 27, 90, 181, 244, 272, 335 hoặc 426 ngày. Như vậy, hai thứ sáu ngày 13 gần nhất có thể cách nhau hơn một năm. Đó chính là trường hợp 13-8-1999 và 13-10-2000.

 

Theo Kinh Thánh, Chúa Jésus bị đóng đinh trên thập tự giá vào thứ sáu. Hơn nữa, bữa ăn cuối cùng của Chúa với các môn đồ có đúng 13 người. Việc này thường được xem là nguồn gốc việc kiêng sợ thứ sáu ngày 13.

 

 

 

 

lastsupperdavinci.jpg

Kiệt tác Bữa tiệc cuối cùng của Léonardo da Vinci

 

 

Ở Ý, số 17 được gắn với sự rủi ro chứ không phải số 13. Còn ở Trung Quốc, con số này là 4 vì được phát âm gần giống với “tử” nghĩa là chết. Ở châu Mỹ Latin, ngày kiêng cữ lại là thứ ba 13.

 

 

Về mặt thống kê, hiện chưa có dữ liệu đáng tin cậy nào để gán cho thứ sáu ngày 13 với “may mắn” hay “rủi ro” theo một nghĩa nào đó. Chẳng hạn, xác suất trúng lô tô ở Pháp vào thứ sáu ngày 13 cũng giống với những ngày khác và xấp xỉ với 1/14.000.000. Xác suất nhỏ bé này không có nghĩa là bạn không thể trúng lô tô và không hề ngăn cản người chơi lô tô nuôi hi vọng!

 

VMF




#465220 Chúc mừng ngày 20/11/2013

Gửi bởi Ban Biên Tập trong 19-11-2013 - 12:08

Topic này dùng để các mem gửi lời chúc đến các thầy cô nhân ngày 20/11


a


#388877 Brachistochrone – Đường đoản thời của John Bernoulli

Gửi bởi Ban Biên Tập trong 21-01-2013 - 20:51

Rất có thể tiêu đề bài viết này đã làm bạn cảm thấy có đôi chút sợ hãi. Ấy vậy mà tôi dám cá rằng các bạn sắp được xem sau đây một trong những chứng minh toán học tuyệt diệu và đặc biệt nhất… Một trong những nguyên lý đẹp nhất của cơ học cổ điển là: Nguyên lý tác dụng tối thiểu. Mà nếu nói một cách nôm na là ” tự nhiên luôn thực hiện mọi việc một cách hết sức tiết kiệm và dè sẻn“. Nói riêng khi ta xét đến hành trình của một tia sáng, nó luôn luôn chọn con đường nào có thời gian đi ngắn nhất. Bây giờ tôi xin được giới thiệu một bài toán cũng tìm con đường đi có thời gian ngắn nhất, chỉ khác một điều: đó không phải là một bài toán quang học nữa, mà lại thuần túy là một bài toán cơ học!

Vào tháng 6 năm 1696, John Bernouilli gửi một lời thách thức đến cho toàn giới Toán học thời bấy giờ (chủ yếu là gửi đến ông anh trai James Bernouilli ) bằng bài toán sau đây:

“Tìm đường cong nối 2 điểm A và B ( trong đó B nằm thấp hơn A và cả hai không cùng nằm trên đường thẳng đứng) sao cho một chất điểm chuyển động không ma sát dưới tác động của trọng lực khởi đầu từ A dọc theo đường cong đó sẽ đến B sớm nhất?”.

Nói cho dễ hiểu là bạn phải làm cái máng trượt có hình thù thế nào để hòn bi lăn đến đích sớm nhất.
Hình đã gửi
Lần này chắc các bạn không nghĩ quỹ đạo cần tìm là đoạn thẳng nối 2 điểm A, B nữa chứ ? Thật ra thì bài toán này không phải là mới. Trong một cuốn sách của mình xuất bản 1638, Galileo cũng đã đề cập đến bài toán này và chứng minh được rằng quỹ đạo là cung tròn thì nhanh hơn quỹ đạo thẳng. Tuy vậy sự lựa chọn đường đi là cung tròn của ông không phải là lời giải đúng.
Hình đã gửi
Về phần mình, John Bernouilli đã đặt tên cho đường cong ngắn nhất đó là ‘Brachistochrone’, mà ông đã nối với nhau hai từ Hi Lạp: ‘Brachistos’ nghĩa là ngắn nhất và ‘chronos’ nghĩa là thời gian. Một cách ‘Việt Nam’ chúng ta có thể gọi nó là ‘đường đoản thời’. Như ta đã biết: trong toán học một hàm số có đồ thị là một đường cong nào đó, và ngược lại nói chung mỗi đường cong sẽ có thể là đồ thị của một hàm số nào đó. Thay vì tìm đường cong, chúng ta sẽ tìm hàm số nhận nó làm đồ thị. Đầu tiên chúng ta đặt đường cong nối từ A đến B vào một hệ tọa độ nhận A làm gốc với trục tung hướng thẳng đứng xuống dưới như sau:
Hình đã gửi
Vì chất điểm chuyển động dưới tác dụng của trọng lực nên ta ‘dễ dàng’ xác định được vận tốc của chất điểm tại điểm C có tung độ $y$ nào đó sẽ là: $v=\sqrt{2gy}$. Xin các bạn yên tâm là tôi sẽ không bắt chước theo các giảng viên đại học bây giờ với các điệp khúc: ” Ồ! dễ thấy rằng… hay không mấy khó khăn ta sẽ…”. Thật ra nhờ đi dạy mà tôi biết rằng trong các trường hợp mà Thầy giáo còn ’khó thấy’ thì dễ dàng nhất là cứ nói: “dễ thấy rằng…” Ngụ ý: chỉ có khờ mới không thấy, mà cuộc đời đâu có ai chịu nhận mình khờ …Trở lại vấn đề, vận tốc tại điểm C có tung độ $y$ sẽ được tìm ra bằng định luật bảo toàn cơ năng (chú ý rằng giả thiết bài toán đã bỏ qua ma sát nên cơ năng bảo toàn):

Đầu tiên để cho đơn giản ta sẽ chọn gốc thế năng tại điểm C. Vì tại A vật không có vận tốc đầu nên cơ năng sẽ chỉ gồm thế năng tại đó: $E=mgy$ (ở đây thế năng sẽ được tính theo độ cao tính từ điểm A xuống gốc thế năng C (tưởng tượng như là mặt đất), mà độ cao này lại chính là $y$. Mặt khác cơ năng tại C chỉ bao gồm động năng ( C là gốc nên thế năng bằng 0): $E=\frac{1}{2}mv^2$. Do cơ năng bảo toàn nên ta có:
$$\frac{1}{2}mv^2 = mgy \Rightarrow v^2 = 2gy \Rightarrow v = \sqrt{2gy}$$
Vận tốc này không hề phụ thuộc vào quỹ đạo của đường cong mà chỉ phụ thuộc vào tung độ y của điểm C đang xét. Và đến đây là điểm quan trọng nhất: Chúng ta chia mặt phẳng chứa đường đi của chất điểm thành vô số những dải lớp mỏng nằm ngang.
Hình đã gửi
Chất điểm sẽ lần lượt đi qua hết lớp này đến lớp khác. Và như trên đã nói vận tốc của nó không phụ thuộc vào quỹ đạo đường đi mà chỉ phụ thuộc vào dải lớp mà nó đi qua ( phụ thuộc vào tung độ y)…Các bạn đã thấy tình huống này ở đâu đó chưa? Đúng vậy! Ánh sáng cũng đã gặp tình huống này khi nó đi từ môi trường không khí vào môi trường nước, hai môi trường mà ở trong đó nó chuyển động với vận tốc khác nhau. Và bởi vì ánh sáng luôn đi theo con đường nhanh nhất nên chúng ta sẽ khôn ngoan đi theo ánh sáng, tức là tuân theo định luật khúc xạ. Bài toán cơ học lúc đầu đã bị chuyển thành một bài toán quang học mất rồi! Thật là một ý tưởng tuyệt diệu.

Trên cơ sở đó chúng ta sẽ nhìn bài toán ban đầu trong một bối cảnh khác: ở đó, có một tia sáng muốn đi từ A đến B. Nó phải đi qua một môi trường biến đổi liên tục mà vận tốc của nó thay đổi theo công thức $v=\sqrt{2gy}$. Vậy nó sẽ đi theo đường nào? Và tất nhiên con đường mà ánh sáng sẽ đi chính là con đường ta cần tìm. Đến đây, dù rằng vẫn còn nhiều việc phải làm nhưng bài toán mới này đã đơn giản hơn bài toán ban đầu biết dường nào rồi!

Gọi $v, v', v'', v''', ...$ lần lượt là vận tốc của ánh sáng trong từng lớp liên tiếp, và $\alpha, \alpha ', \alpha '', \alpha ''', ...$ là các góc tới tương ứng của tia sáng khi đến gặp các mặt phân cách (góc tạo bởi tia sáng với trục thẳng đứng). Theo định luật khúc xạ ánh sáng ta có:
$$\frac{sin\alpha}{v} =\frac{sin\alpha '}{v'} = \frac{sin\alpha''}{v''} = \frac{sin\alpha'''}{v'''} = ...$$
Và bởi vì $v$ nhận các giá trị liên tục theo độ sâu (cũng có nghĩa là các dải lớp môi trường của ta có thể cho mỏng đi một cách tùy ý) ta sẽ có: $\frac{sin\alpha}{v} = const \textbf{ (*)}$ dọc theo đường đi của ánh sáng.

Tiếp theo sẽ là một điểm khá tinh tế (dù không khó khăn để tưởng tượng). Đối với các lớp chúng ta đang xét, chẳng hạn lớp trên cùng có vận tốc $v$, mình sẽ cho bề dày của lớp này tiến dần về 0:

Hình đã gửi
Có nghĩa là cho điểm $P$ tiến dần về điểm $R$, lúc này đoạn $PR$ sẽ tiến dần tới tiếp tuyến của đường cong. Như vậy góc $\alpha$ thực chất sẽ là góc tạo bởi tiếp tuyến của đường cong tại $R$ và trục thẳng đứng.
Hình đã gửi
Ta gọi $\beta$ là góc tạo bởi tiếp tuyến tại $R$ và trục nằm ngang. Mọi người học lớp 7 rồi chắc là còn nhớ $\tan\beta$ chính là hệ số góc của tiếp tuyến của đường cong tại $R$. Mặt khác, nếu bạn đã học lớp 11 rồi thì sẽ biết luôn rằng hệ số góc của tiếp tuyến thì bằng với đạo hàm của hàm số tại đó, tức là: $\tan\beta = y'$. Mặt khác, hiển nhiên là:
$$\alpha + \beta = 90^o \Rightarrow \sin \alpha = \cos \beta = \frac{1}{\sqrt{1+y'^2}}$$.
Cái này được suy ra từ công thức lượng giác cơ bản: $1 + \tan^2\beta = \frac{1}{\cos^2\beta}$. Kết hợp với $(*)$ và và nhớ lại rằng $v = \sqrt{2gy}$ ta được:
$$const = \frac{\sin \alpha}{v} = \frac{1}{\sqrt{1+y'^2}\sqrt{2gy}} \Rightarrow y(1+y'^2) = const = c$$
Cuối cùng thì ta cũng đi đến được phương trình này (là phương trình mà hàm số $y$ của chúng ta phải thõa mãn):
$$y(1+y'^2) = c \textbf{ (**)}$$
Bấy lâu nay mọi người vẫn thường nói: “Toán học là ông hoàng mà cũng là người đầy tớ của khoa học”. Các bạn có biết vì sao không? Là ông hoàng thì xin không giải thích thêm, còn làm “đầy tớ” thì đây là một minh chứng rõ nét. Vật lý chỉ cần dựa vào lý thuyết của mình rút ra phương trình mà đại lượng nào đó phải thõa mãn. Công việc còn lại giao cho toán học xử lý, nó sẽ giải và tìm xem hàm số ấy là gì?

Tất nhiên phương trình $(**)$ là một phương trình vi phân và ẩn của nó làm một hàm số (hàm số chúng ta cần tìm). Nhiều bạn học sinh vẫn chưa biết giải thì cũng đừng bận tâm, vì nói cho cùng thì những phương trình như vậy đều đã có cách giải sẵn. Và thậm chí bạn có thể tìm ra nghiệm của nó chỉ với một phần mềm tính toán mạnh (Maple chẳng hạn). Hàm số tìm được có đồ thị là một đường cong với tên gọi: Cycloid. Tôi xin phép chỉ mô ta đường cong này một chút mà không viết ra đây công thức hàm số chúng ta tìm được. Nó cũng không hẳn phức tạp, chỉ là nó được viết dưới dạng tham số, mà tôi không muốn bài viết của mình dài thêm nữa.

Cùng tìm hiểu một chút về đường cong Cycloid, bạn có thể dễ dàng tưởng tượng ra hình ảnh của nó. Thật vậy, hãy gắn một điểm sáng vào một cái bánh xe đạp rồi lăn nó đi trong bóng tối. Quỹ đạo của điểm sáng đó cũng chính là hình ảnh của đường cong Cycloid.
Hình đã gửi
Và bây giờ là lúc ta tận hưởng thành quả: Đây là tính toán mô phỏng chuyển động của chất điểm theo những quỹ đạo khác nhau. Các bạn hãy xem:
Hình đã gửi
Các bạn chắc biết đâu là đường Cycloid của mình rồi chứ? Lời cuối cùng xin cảm ơn John Bernouilli với chứng minh tuyệt diệu này. Tôi sẽ không quá cường điệu khi nói rằng việc tìm ra chứng minh đó là một nghệ thuật! Chúng ta đã trải qua hai ’ngữ cảnh’ rất khác nhau: Đầu tiên là cơ học, rồi thoắt một cái nó biến thành bài toán quang học. Và quả thật các bạn đã thấy ở đâu một sự liên hệ độc đáo và thú vị đến như vậy hay chưa?

Theo Ngô Minh Đức


#373185 Hình học Tĩnh và động

Gửi bởi Ban Biên Tập trong 27-11-2012 - 22:28

HÌNH HỌC TĨNH VÀ ĐỘNG

Lê Bá Khánh Trình (Đại học KHTN, ĐHQG Tp. Hồ Chí Minh)

1. HÌNH HỌC TĨNH HAY ĐỘNG

Trong bài này, tôi muốn trình bày một đôi điều riêng tư về môn hình học phổ thông (hay còn được gọi là hình học sơ cấp) dưới hai cách nhìn có phần nào khác biệt nhau. Trước hết, thông dụng hơn cả là cách nhìn của một người quan tâm đến việc giải các bài toán hình học. Cách nhìn này thường yêu cầu xem xét, phân loại các bài toán khác nhau, trình bày kinh nghiệm giải quyết chúng và tìm ra các mối liên quan giữa chúng với các bài toán đã biết. Cách nhìn này thường được quan tâm hàng đầu và thường là nội dung chính trong các bài viết, các tài liệu về toán phổ thông. Bên cạnh đó, tôi cũng muốn trình bày các vấn đề ở đây dưới một cách nhìn khác, cách nhìn của người muốn tìm tòi, phát hiện ra các bài toán mới, những bài toán không chỉ mới về nội dung mà còn có tác dụng tích cực trong việc rèn luyện tư duy và các kỹ năng cần thiết của người học, đặc biệt là đối với những học sinh giỏi. Đây là công việc đòi hỏi ở chúng ta nhiều công phu không kém gì công việc giải quyết các bài toán. Tuy nhiên, ở nước ta dường như công việc này còn chưa được quan tâm đúng mức. Đây đó, được ưa chuộng hơn cả vẫn là sử dụng các bài toán hay, mẫu mực đã có hoặc tận dụng các đề toán mới được công bố ở các nước khác. Cách làm này khá tiện lợi, hợp lý và hiệu quả nhưng thực tế có hai nguy cơ:
∙ Một là, nếu sử dụng các bài toán đã được công bố trong các kỳ thi, việc đánh giá sẽ thiếu công bằng và chính xác;
∙ Hai là, đáp án của nhiều bài toán do vô tình hay hữu ý, đã ít nhiều bị biến dạng. Điều này có thể làm cho cách trình bày trở nên ngắn gọn hơn nhưng đồng thời cũng đã làm mất đi những ý tưởng trong sáng và tự nhiên ban đầu khi những bài toán đó được xây dựng nên. Vì thế, nếu sử dụng lại các đáp án một cách máy móc, thiếu sự biên tập cần thiết thì rất có thể chúng sẽ có tác dụng tiêu cực đến việc rèn luyện tư duy của người học.

Với những suy nghĩ đó, tôi nghĩ chắc cũng đã đến lúc chúng ta cần tăng cường sự quan tâm và đầu tư nhiều công sức hơn nữa cho công việc “sáng tác” này. Một công việc không dễ dàng nhưng chắc chắn sẽ rất thú vị và bổ ích. Bây giờ, đã đến lúc đi thẳng vào chủ đề của bài này: Hình học tĩnh hay động? Nếu chỉ nhìn các bài toán mà chúng ta vẫn thường giải quyết hoặc tìm tòi thì hình học vừa tĩnh lại vừa động. Hình học tĩnh trong những bài toán mà ở đó, các yếu tố như điểm, đường thẳng, đường tròn,... đều không thay đổi và yêu cầu đặt ra ở đây thường là chứng minh các tính chất hình học hoặc tính toán các đại lượng nào đó trong hình vẽ đã cho. Còn hình học sẽ động trong những bài toán mà ở đó, bên cạnh các yếu tố cố định, không thay đổi có 1 vài yếu tố thay đổi và yêu cầu ở đây thường là tìm quĩ tích, tìm các điểm cố định hoặc tìm giá trị lớn nhất, nhỏ nhất của một đại lượng hình học. Tuy nhiên, đây chỉ là cái nhìn ban đầu. Trên quan điểm của những người mong muốn đi tìm lời giải cho các bài toán khó và cả trên quan điểm của những người mong muốn phát hiện ra những bài toán hình học mới, theo tôi, hình học luôn luôn cần vận động, vận động ngay cả trong những bài toán mà các yếu tố được cho đều cố định, không đổi. Bởi vì chính cách nhìn, cách tư duy trong các yếu tố của hình vẽ không ngừng biến động, tuơng tác, thậm chí toàn bộ cả hình vẽ đều không thay đổi sẽ giúp chúng ta tìm ra đúng những lời giải đẹp nhất và phản ánh trọn vẹn nhất bản chất hình học của một bài toán.




2. ĐỘNG TRONG BIẾN HÌNH

Một trong những công cụ quan trọng hàng đầu để thực hiện việc biến đổi các yếu tố trong một hình chính là phép biến hình. Không phải ngẫu nhiên mà hiện nay, những lời giải hay nhất của nhiều bài toán hình học cũng như rất nhiều phát hiện hình học thú vị thường nhận được trên cơ sở vận động ý tưởng và kỹ thuật của các phép biến hình.

Thế nhưng để có thể vận dụng chúng một cách hiệu quả, trước hết phải có được một nền tảng tương đối vững chắc về biến hình mà cụ thể là phải nắm bắt được một vài mệnh đề quan trọng và làm quen được với một số tình huống tiêu biểu cho việc thực hiện các động tác biến hình hợp lý.

Vậy đó là những mệnh đề nào, những tình huống nào? Trong khuôn khổ bài này, tôi chỉ xin phép trình bày những gì liên quan đến phép quay, một loại phép biến hình tuy đơn giản nhưng lại có mức độ áp dụng cao và mang lại rất nhiều kết quả phong phú. Tương tự, không khác biệt với phép quay bao nhiêu là phép vị tự quay. Thông thường, phép vị tự quay đem lại các kết quả tổng quát hơn và nâng cao độ phức tạp của bài toán mà vẫn giữ nguyên ý tưởng ban đầu của phép quay.

Nhưng trước khi phát biểu ra đây các mệnh đề, tình huống cần thiết được nhắc ở trên, xin phép được nói qua một chút cái gọi là “cảm hứng” thúc đẩy tôi viết ra những dòng này. “Cảm hứng” đó nảy sinh từ việc xem xét giáo trình Hình học nâng cao lớp 11 vừa được đưa vào giảng dạy từ vài năm học vừa qua, trong đó điểm đáng lưu ý nhất là phần các phép biến hình được trình bày đầy đủ hơn và đặc biệt là đã được phân bố ngay vào đầu năm học (trước đây, phần này chỉ được giảng dạy vào cuối năm lớp 10). Rõ ràng, với sự thay đổi này, hội đồng biên soạn sách giáo khoa cho thấy ý định rất nghiêm túc của mình là tăng cường hơn nữa sự chú ý cho phần các phép biến hình và đây thực sự là điều rất nên làm.

Các phép biến hình chính là mảng kiến thức mà ở đó, học sinh có thể làm được với những ý tưởng và những kỹ năng thích hợp nhất cho việc tiếp thu các kiến thức của toán học hiện đại. Những ý tưởng và những kỹ năng đó là gì? Đó là ý tưởng ánh xạ rất rõ nét trong cách trình bày và hệ thống các phép biến hình. Đó là ý tưởng phân loại và mô tả đầy đủ các lớp phép biến hình (mà tiêu biểu nhất là các phép dời hình). Và tất nhiên, quan trọng hơn cả là qua việc vận dụng các phép biến hình để giải toán, tư duy hình học của học sinh sẽ được nâng lên ở một cấp độ mới. Thay vì chỉ biết tính toán và so sánh các đại lượng hình học (góc, độ dài, diện tích,... ) để từ đó đi đến một chứng minh như trước đây, nay với việc sử dụng các phép biến hình, các em sẽ được tập quan sát những vận động, những tương tác giữa các yếu tố, những cấu trúc tiềm ẩn trong một hình vẽ để rồi từ đó rút ra được những chứng minh, những kết luận sâu sắc, nêu bật toàn diện bản chất của hình vẽ đó.

Những ý định như vậy là rất đúng đắn và chắc cũng đã được hội đồng biên soạn sách giáo khoa đem ra cân nhắc kỹ lưỡng trước khi quyết định việc phân bố lại chương trình sách giáo khoa nâng cao về hình học. Chỉ tiếc một điều, theo nhận xét chủ quan của tôi, là nội dung trình bày trong sách giáo khoa lớp 11 có lẽ vẫn còn chưa đủ để học sinh rèn luyện, nắm bắt và vận dụng công cụ biến hình ở mức độ cần thiết, ít ra là chưa cho phép các em làm quen được với ba ý tưởng quan trọng và bổ ích được kể ra ở trên.
Vậy nên cần bổ sung những điều gì? Xin điểm qua một vài điều tôi cho là quan trọng nhất và nhân tiện, đây cũng chính là trả lời cho câu hỏi đặt ra ở đầu phần này. Đó là phát biểu các mệnh đề, các tình huống chính mà bất cứ ai khi học các phép toán biến hình (cụ thể là phép quay) đều phải biết để có thể vận dụng thực sự tốt công cụ này.

2.1. Sự tồn tại của phép quay. Trước hết, để giúp cho học sinh hiểu rõ và tự tin hơn khi sử dụng các phép biến hình, nên trang bị cho các em các mệnh đề về tồn tại duy nhất của một phép biến hình trong những tình huống đơn giản và thông dụng nhất. Đối với phép quay, mệnh đề sau đáp ứng đủ các yêu cầu đó.

Mệnh đề 2.1. Cho hai đoạn thẳng $AB$ và $A'B'$ sao cho $AB = A'B'$ và $\overrightarrow{AB}\neq \overrightarrow{A'B'}$ Lúc đó, tồn tại duy nhất một phép quay $R$ biến tương ứng $AB$ thành $A'B'$.


phepquay1.png


Mệnh đề này cho phép ta chỉ cần quan sát thấy có hai đoạn thẳng bằng nhau là có thể liên tưởng ngay đến một phép quay và sẵn sàng vận dụng nó nếu có thêm các điều kiện thích hợp chứ không phải chờ đến khi có được hai tam giác, hai hình bằng nhau mới bắt đầu nghĩ đến phép quay. Ngoài ra, mệnh đề này còn là cơ sở để mô tả đầy đủ các phép dời hình (sẽ đề cập ở dưới). Tuy nhiên, nó chỉ có ý nghĩa giúp ta làm quen với tình huống. Muốn mang lại hiệu quả thực sự phải bổ sung thêm một ít về việc xác định phép quay tồn tại nói trên.

Mệnh đề 2.2 (Mệnh đề 1 bổ sung). Phép quay $R$ có góc quay $\alpha = (\overrightarrow{AB}, \overrightarrow{A'B'})$ và tâm $O$ đồng thời nằm trên các trung trực của $AA',BB'$ cũng như các cung tròn (đơn) chứa các điểm nhìn đoạn $AA',BB'$ dưới một góc có hướng bằng $\alpha$.


Bổ sung này cho ta một cái nhìn khá toàn diện về tình huống đang xét (xem hình vẽ); nhưng để có được sự quan sát đầy đặn và sâu sắc hơn nữa, cần trang bị thêm:

Mệnh đề 2.3. Ta giữ các giả thiết như trong các Mệnh đề 1 và 2.
(1) Giả sử các đường thằng $AB$ và $A'B'$ cắt nhau tại điểm $P$. Khi đó, các tứ giác
$AA'OP$ và $BB'OP$ nội tiếp;
(2) Giả sử các đường thẳng $AA'$ và $BB'$ cắt nhau tại điểm $Q$. Khi đó, các tứ giác
$ABOQ$ và $A'B'OQ$ nội tiếp.

Các mệnh đề này rõ ràng là chứng minh không khó (nên xin bỏ qua ở đây). Còn lợi ích mà chúng có thể mang lại thì lại khá phong phú. Xin bắt đầu bằng một bài tập khá quen thuộc trong đó việc vận dụng ý tưởng biến hình là rất tự nhiên và đơn giản.

Ví dụ 2.4. Cho tam giác $ABC$ cân tại $A$. Trên cạnh $AB$ và $AC$ lần lượt lấy các điểm
$M, N$ sao cho $AM = CN$. Chứng minh đường tròn ngoại tiếp tam giác $AMN$ luôn đi qua một điểm cố định khác $A$.


PHEPQUAY2.png


Lời giải. Để giải, ta xét phép quay $R$ biến đọan thẳng $AM$ tương ứng thành đoạn thẳng $CN$.
Tâm quay $O$ theo mệnh đề 2 là giao điểm của trung trực $AC$ và cung tròn quĩ tích những điểm $K$ sao cho:

$$\left (\overrightarrow{KA}, \overrightarrow{KC} \right ) = \left (\overrightarrow{AM}, \overrightarrow{CN} \right )$$

nên tâm quay $O$ cố định.
Cuối cùng, do $AM$ và $CN$ cắt nhau tại $A$ nên theo Mệnh đề 3, tứ giác $MNAO$ nội tiếp. Vậy đường tròn ngoại tiếp tam giác $AMN$ đi qua điểm $O$ cố định.

Bài tập này rất thích hợp cho việc làm quen với các ứng dụng của phép quay. Nó chỉ có một khiếm khuyết là nếu tam giác $ABC$ cân thì điểm $O$ cần tìm chính là tâm đường tròn ngoại tiếp tam giác $ABC$. Do đó, nhiều học sinh có thể mày mò, dự đoán và chứng minh kết quả trên mà không cần sử dụng phép quay. Thực ra, để khắc phục điều này, có thể xem tam giác $ABC$ không cân và còn tổng quát hơn là bài tập sau mà cách giải không có gì thay đổi.

Ví dụ 2.5. Trên 2 tia $Ox$ và $Oy$ của góc $Oxy$, cho 2 điểm $A,B$. Gọi $M,N$ là 2 điểm thay đổi trên $Ox, Oy$ sao cho $AM=BN$ ($M$ khác phía $O$ đối với điểm $A$, còn $N$ cùng phía $O$ đối với điểm $B$). Chứng minh rằng đường tròn ngoại tiếp tam giác $OMN$ luôn đi qua một điểm cố định khác $O$


PHEPQUAY3.png


Nếu bổ sung vào bài tập này thêm một vài yếu tố với những mối quan hệ tương tự (chẳng hạn lấy thêm các điểm $P,Q$ trên $Ox,Oy$ cũng với tính chất $AP=BQ$ để phép quay được xét cũng biến $P$ thành $Q$) và thay đổi chút ít cách phát biểu cũng như vận dụng tính chất còn lại (tính chất 2) của Mệnh đề 3. Ta nhận được:

Bài toán. Cho tứ giác $ABC$ có $AB=CD$ và các điểm $M,N$ trên $AB,CD$ sao cho $AM=DN$. Giả sử đường thẳng $MN$ cắt $AD$ và $BC$ lần lượt tại $P,Q$. Chứng minh rằng tồn tại một điểm $O$ có cùng phương tích với tất cả bốn đường tròn ngoại tiếp các tam giác $PAM,PDN,QBM,QCN$


phepquay4.png


Lời giải. Gọi $O$ là tâm của phép quay $R$ biến $AB$ tương ứng thành $CD$ và $M$ thành $N$. Theo mệnh đề 3 (tính chất 2), các tứ giác $AMOP,ANOP,BMOQ,CNOQ$ đều nội tiếp. Vậy $O$ nằm trên bốn đường tròn nội tiếp các tam giác $PAM,PDN,QBM,QCN$
nên $O$ có cùng phương tích đối với các đường tròn này.

2.2. Tích của hai phép quay. Điều cần bổ sung thứ hai liên quan đến bản chất ánh xạ của các phép biến hình. Một khi đã định nghĩa chúng như các ánh xạ thì lẽ tự nhiên cũng cần phải đề cập đến tích của hai phép biến hình. Vậy tích của hai phép quay là gì?

Mệnh đề 2.6. Cho hai phép quay $R(O_1; \alpha _1), R(O_2; \alpha _ 2)$. Nếu $\alpha _1 + \alpha _2 = 2k\pi$ thì tích $R = R_2.R_1$ cũng là một phép quay với góc quay $\alpha = \alpha _1 + \alpha _2$. Tâm $O$ của phép quay này được xác định từ điều kiện sau
$$\left (\overrightarrow{O_1O}, \overrightarrow{O_1O_2} \right ) = \frac{\alpha_1}{2};\left (\overrightarrow{O_2O_1}, \overrightarrow{O_2O} \right ) = \frac{\alpha_2}{2}$$


phepquay5.png


Chứng minh. Việc $R$ là một phép quay có thể suy ra ngay từ Mệnh đề 1. Còn tâm $O$ chính là điểm bất động duy nhất qua tích
$R = R_2.R_1$ nên nếu chọn $O$ như trên
và lấy $O’$ đối xứng với $O$ qua $O_1O_2$ thì ta
có $R_1(O)=O’$ và $R_2(O’) = O$. Suy ra
$R(O) = O$. Vậy điểm $O$ xác định với điều kiện trên chính là tâm quay.

Bài tập sau có thể xem là ứng dụng mẫu mực việc vận dụng tích 2 phép quay.

Ví dụ 2.7. Bên ngoài tam giác $ABC$ và trên các cạnh dựng các tam giác $BCA_1,CAB_1,ABC_1$ cân lần lượt tại $A_1,B_1,C_1$ với góc $BA_1C = 160^o$ và các góc $CB_1A=AC_1B= 100^o$. Tính góc $B_1A_1C_1$


phepquay6.png


Lời giải. Nhận xét rằng:
$$R(A_1;-160^o)=R(B_1;100^o).R(C_1;100^o)$$
Như vậy, theo tính chất tâm của tích hai phép quay thì góc $B_1A_1C_1 = 80^o$


Tất nhiên, với đề bài như trên, một số học sinh vẫn có thể đi “tính” góc $B_1A_1C_1$ với một khối lượng tính toán hết sức cồng kềnh và với kỹ thuật tính toán đáng nể. Nếu bây giờ biến tấu bài tập này đi một chút bằng cách cất đi điểm “mấu chốt” $A_1$ và gắn thêm tính di động cho các điểm $B_1,C_1$, ta nhận được phương án sau:


Bài toán. Cho tam giác $ABC$ nội tiếp đường tròn $(O)$ có $B,C$ cố định, còn $A$ thay đổi trên $(O)$. Bên ngoài tam giác, trên các cạnh $AB,AC$ dựng các tam giác $ABC_1,ACB_1$ với $\widehat{AC_1B}=\widehat{AB_1C}=100^o$. Chứng minh rằng trung trực của $B_1C_1$ luôn đi qua một điểm cố định.

Rõ ràng điểm cố định cần tìm chính là điểm $A_1$ trong bài tập trên nay đã được “giấu” đi. Và chính vị trí không dễ đoán của $A_1$ đã làm cho bài toán trở nên vô cùng khó khăn cho những ai chưa nắm được ý tưởng về tích của hai phép quay.

2.3. Về các phép dời hình khác. Để kết thúc phần này, xin nêu ra điều cần bổ sung cuối cùng để cho nội dung về phép biến hình được cân đối, hoàn chỉnh. Chúng ta biết rằng lớp các phép biến hình được trình bày đầy đủ nhất chính là lớp các phép dời hình. Chúng có thể được mô tả rất trọn vẹn thông qua các phép dời hình cơ sở là tịnh tiến, quay và đối xứng trục. Vậy nên chăng sau khi đã học xong các phép biến hình cụ thể này, chúng ta sẽ khái quát bằng khái niệm các phép dời hình và kết thúc bằng một mệnh đề mô tả đầy đủ lớp các phép dời hình để làm sáng tỏ bản chất khá đơn giản của chúng. Đây thường là sơ đồ mẫu mực khi trình bày về một lớp các phép biến đổi nào đó trong các lĩnh vực khác của toán học.
Mệnh đề mô tả các phép dời hình ở đây rất gọn, đơn giản và có thể suy ra

trực tiếp từ Mệnh đề 1 ở trên. Nhưng trước khi phát biểu nó, theo tôi nên phân loại các phép dời hình thành các phép dời hình thuận (là các phép dời hình bảo toàn định hướng) và các phép dời hình ngược (thay đổi định hướng). Điều này cũng gần giống như việc phân biệt hai tam giác bằng nhau thuận và bằng nhau nghịch mà học sinh đã rất quen thuộc. Việc phân loại các phép dời hình như vậy sẽ không gây ra khó khăn nào mà trái lại, nó còn có thể giúp học sinh hiểu và cảm nhận rõ ràng hơn về định hướng (cụ thể là chiều “quay” của một tam giác) trong các phép biến hình.
Đối với các phép dời hình thuận (quan trọng nhất và được xem xét kỹ lưỡng nhất) ta có sự mô tả đầy đủ sau:

Mệnh đề 2.8. Một phép dời hình thuận chỉ có thể là một phép tịnh tiến hoặc một phép quay.

Đối với các phép dời hình nghịch thì khó khăn hơn một chút:

Mệnh đề 2.9. Một phép dời hình nghịch có thể được biểu diễn như là tích một phép tịnh tiến với một phép đối xứng trục.


Trong phần bài tập của bộ sách giáo khoa Hình học nâng cao lớp 11, dạng tích này cũng được xét đến và được gọi là phép “đối xứng trượt”. Theo tôi, Mệnh đề 2.9 có thể không nhất thiết phải trình bày hoặc chỉ cần nhắc qua và đưa ra như một bài tập. Nhưng Mệnh đề 2.8 thì nên phát biểu như một lời đúc kết của phần các phép dời hình để sao cho khi học xong phần này, học sinh có cảm giác nắm bắt trọn vẹn, rõ ràng, không còn chút gì mơ hồ về các phép dời hình.

(còn nữa)




#340954 Chuyên đề Hệ phương trình

Gửi bởi Ban Biên Tập trong 28-07-2012 - 00:00

$ \bullet $ Mở rộng cách nhìn về hệ đối xứng kiểu II.

Trước hết, hãy xem xét cách giải hệ phương trình sau:

Ví dụ 23: $\left\{\begin{matrix} x^3+y^2=1 & \\y^3+x^2=1 & \end{matrix}\right.$

Bài giải: Trừ theo vế 2 phương trình của hệ ta thu được:
\[{x^3} - {y^3} + {y^2} - {x^2} = 0 \Leftrightarrow \left( {x - y} \right)\left[ {{x^2} + xy + {y^2} - \left( {x + y} \right)} \right] = 0\]
$$ \Leftrightarrow \left[ \begin{array}{l}x = y\\ {x^2} + xy + {y^2} = x + y \end{array} \right.$$


Trường hợp: $x = y$ thì thế và giải phương trình:
$$x^3+x^2-1=0\Leftrightarrow x=y=\frac{1}{3}\left [ \sqrt[3]{\frac{25}{2}-\frac{3\sqrt{69}}{2}}+\sqrt[3]{\frac{25}{2}+\frac{3\sqrt{69}}{2}}-1 \right ]$$
Cộng theo vế 2 phương trình và kết hợp với $x^2+xy+y^2=x+y$ ta được hệ đối xứng loại I:
$$\left\{\begin{matrix} x^2+xy+y^2=x+y & \\x^3+y^3+x^2+y^2=2 & \end{matrix}\right.$$
$$\Leftrightarrow \left\{\begin{matrix} (x+y)^2-xy=(x+y) & \\(x+y)^3-3xy(x+y)+(x+y)^2-2xy=2 & \end{matrix}\right.$$
Đặt $S=x+y$, $P=xy$, hệ trở thành:

\[\left\{ \begin{array}{l} {S^2} - P = S\\{S^3} - 3SP + {S^2} - 2P = 2\end{array} \right. \Leftrightarrow \left\{ \begin{array}{l}{S^3} + {S^2} - 2 - (3S + 2)({S^2} - S) = 0\\P = {S^2} - S\end{array} \right.\]

\[\left\{ \begin{array}{l} (S - 1)({S^2} + 1) = 0\\ P = {S^2} - S \end{array} \right. \Leftrightarrow \left\{ \begin{array}{l} S = 1\\ P = 0 \end{array} \right. \Leftrightarrow \left\{ \begin{array}{l} x = 1,y = 0\\ x = 0,y = 1 \end{array} \right.\]
Vậy hệ đã cho có 3 nghiệm như trên.

Hình đã gửi

$ \bullet $ Qua bài giải trên, hẳn chúng ta nhận rõ vai trò của việc kết hợp “cộng” và “trừ” để đưa đến hpt đối xứng kiểu I (đây là một hướng nhìn mới). Việc làm này hoàn toàn có cơ sở. Hãy xem lại câu nói: “Cũng như loại I, loại II cũng có “đối xứng” nhưng là đối xứng giữa 2 phương trình chứ không phải là đối xứng trong từng phương trình như kiểu I”. Như vậy, khi cộng theo vế sẽ luôn cho một trong hai phương trình đối xứng kiểu I. Và việc lấy đi nghiệm $(x-y)$ sau khi trừ cũng để lại cho ta 1 phương trình đối xứng nữa.

Ví dụ 24: Giải hệ phương trình: $\left\{\begin{matrix} x^3+y=2 & \\y^3+x=2 & \end{matrix}\right.$

$ \bullet $ Và đôi lúc việc cộng trừ cũng không đem lại cho ta kết quả khả quan:

Ví dụ 25: Giải hệ phương trình: $\left\{\begin{matrix} x+\frac{2xy}{\sqrt[3]{x^2-2x+9}}=x^2+y & \\y+\frac{2xy}{\sqrt[3]{y^2-2y+9}}=y^2+x & \end{matrix}\right.$

Nhận xét: Quả đúng, khi cộng hay trừ ta không thể làm gì với cái căn khủng khiếp kia. Tuy nhiên, một số điểm ta lại thấy rõ và đáng phải nghĩ là trong cái căn bậc 3 kia, có một đẳng thức: $(x-1)^2+8=x^2-2x+9$. Nhẩm nghiệm $x=y=1$ và $\sqrt[3]{8}=2$ (căn đẹp!). Phải liên kết và sử dụng chúng như thế nào?

Bài giải: Cộng theo vế 2 hệ của phương trình: $\frac{2xy}{\sqrt[3]{x^2-2x+9}}+\frac{2xy}{\sqrt[3]{y^2-2y+9}}=x^2+y^2$.

Sử dụng đánh giá:
$$\sqrt[3]{x^2-2x+9}=\sqrt[3]{(x-1)^2+8}\geq 2\Rightarrow \frac{2xy}{\sqrt[3]{x^2-2x+9}}\leq \frac{2xy}{2}=xy$$
Tương tự ta có:
$$\frac{2xy}{\sqrt[3]{y^2-2y+9}}\leq xy\Rightarrow \frac{2xy}{\sqrt[3]{x^2-2x+9}}+\frac{2xy}{\sqrt[3]{y^2-2y+9}}\leq 2xy=x^2+y^2-(x-y)^2\leq x^2+y^2$$
Vậy hệ có nghiệm khi $x=y=1$, thử lại thấy đúng, kết luận nghiệm.

$ \bullet $ 2 kiểu hệ đối xứng I và II là những dạng rất cơ bản. Tuy nhiên, qua “chế biến” của người ra đề thì không thể nói trước được điều gì. Vì vậy, cần có cái nhìn tổng quan, nhìn nhiều khía cạnh, không nên chỉ biết nhìn hình thức rồi rập khuôn lời giải của dạng. Một số ví dụ thêm:

Ví dụ 26: (Thi thử ĐH CĐ THPT Lê Văn Hưu, Thanh Hóa năm 2011)
Giải hệ phương trình: $\left\{\begin{matrix} 2x^2+x-\frac{1}{y}=2 & \\y-y^2x-2y^2=-2 & \end{matrix}\right.$

Ví dụ 27: Giải hệ phương trình: $\left\{\begin{matrix} x^3(2+3y)=1 & \\(y^3-2)x=3 & \end{matrix}\right.$

Ví dụ 28: (Tuyển sinh vào lớp 10 THPT Chuyên Phan Bội Châu 2009-2010)
Giải hệ phương trình: $\left\{\begin{matrix} x^3(2+3y)=8 & \\(y^3-2)x=6 & \end{matrix}\right.$

Ví dụ 29: (Thi ĐH - CĐ khối B 2003)
Giải hệ phương trình: $\left\{\begin{matrix} 3y=\frac{y^2+2}{x^2} & \\3x=\frac{x^2+2}{y^2} & \end{matrix}\right.$

Ví dụ 30: (Olympic 30-4-2010)
Giải hệ phương trình: $\left\{\begin{matrix} \sqrt{x^2+2x+22}-\sqrt{y}=y^2+2y+1 & \\\sqrt{y^2+2y+22}-\sqrt{x}=x^2+2x+1 & \end{matrix}\right.$

Ví dụ 31: (Thi thử ĐH năm 2011, THTT số 379, 2009)
Giải hệ phương trình: $\left\{\begin{matrix} x+\sqrt{x^2-2x+2}=3^{y-1}+1 & \\y+\sqrt{y^2-2y+2}=3^{x-1}+1 & \end{matrix}\right.$

Ví dụ 32: Giải hệ phương trình: $\left\{\begin{matrix} x+\sqrt{x^2-2x+2}=3^{y-1}+1 & \\y+\sqrt{y^2-2y+2}=3^{x-1}+1 & \end{matrix}\right.$

III. MỘT SỐ PHƯƠNG PHÁP MỚI

1. Phương pháp 01: Hằng số = $t$ = ẩn số:

Xem xét cách giải một số ví dụ sau:


Ví dụ 1: (Hệ phương trình TST Nghệ An 2009-2010)
Giải hệ phương trình: $\left\{\begin{matrix} x^2+y^2=\frac{1}{5} & \\4x^2+3x-\frac{57}{25}=-y(3x+1) & \end{matrix}\right.$

Bài giải: Nhân phương trình sau của hệ với $2$ rồi cộng theo vế với phương trình đầu ta được:
$$9{x^2} + {y^2} + 6xy + 6x + 2y = \frac{{119}}{{25}} \Leftrightarrow {(3x + y + 1)^2} = \frac{{144}}{{25}} \Leftrightarrow \left [ \begin{array}{l}3x + y + 1 = \frac{{12}}{5}\\3x + y + 1 = - \frac{{12}}{5}\end{array} \right.$$

Đến đây thì thế vào phương trình ban đầu ta giải phương trình bậc 2 nữa là xong.


Ví dụ 2: Giải hệ phương trình: $\left\{\begin{matrix} (x-y)^2+y=3 & \\x^2+2xy-5y^2-5x+13y=6 & \end{matrix}\right.$

Bài giải: Nhân 3 vào phương trình đầu rồi trừ theo vế với phương trình sau ta được:
$$2{x^2} + 8{y^2} - 8xy + 5x - 10y = 3 \Leftrightarrow 2{(x - 2y)^2} + 5(x - 2y) - 3 = 0 $$

$$\Leftrightarrow (x - 2y + 3)(2x - 4y - 1) = 0 \Leftrightarrow \left[ \begin{array}{l}x - 2y = - 3\\2x - 4y = 1\end{array} \right.$$



Đến đây, thế từng trường hợp rồi thay vào phương trình ban đầu là xong.

$ \bullet $ Nhận xét: Hai bài giải trên thật hay, đơn giản với công việc nhân thêm rồi cộng lại, sau đó phân tích thành nhân tử.


Nhưng! Điều chúng ta băn khoăn và thắc mắc ở đây chính là việc biết phải nhân với con số nào. Đây chính là cơ sở để chúng ta đi đến phương pháp ẩn số $= t =$ hằng số.

$ \bullet $ Như chúng ta đã biết, cái chưa biết chính là ẩn số. Đây cũng vậy, để biết cần nhân với bao nhiêu, ta đưa thêm ẩn $t$ vào. Do đó, hpt của chúng ta đã có đến tận 3 ẩn với chỉ 2 giả thuyết. Như vậy, phải có thêm một cái gì đó ràng buộc. Nó là gì? Quan sát lại 2 ví dụ trên một lần nữa.

$ \bullet $ Phương pháp: Hằng số $= t =$ ẩn số:

- Phạm vi ứng dụng: hệ phương trình 2 ẩn $x$, $y$ có bậc không quá 2.

- Cơ sở phương pháp: giải phương trình bậc 2.


Xét phương trình: $ax^2+bx+c=0$. Có: $\Delta =b^2-4ac$.

Nếu: $\Delta

Nếu $\Delta >0$ phương trình có 2 nghiệm phân biệt.

Đặc biệt: $\Delta =0$ phương trình có 1 nghiệm duy nhất, tức là khi đó phương trình tương đương với: $$a(x+\frac{b}{2a})^2=0$$. Đây chính là cơ sở cơ bản của phương pháp
.
(Bài viết sẽ không trình bày giải hệ phương trình tổng quát mà sẽ thực hiện giải chi tiết những ví dụ cụ thể nhằm tạo cho bạn những tu duy, suy nghĩ mới và tự hình thành cho mình những phương pháp và kĩ năng. Hơn nữa việc trình bày tổng quát khá phức tạp)

Hãy xem xét lại 2 ví dụ trên:

Thay vì nhân vào những con số $2$ như Ví dụ 1, con số $3$ như Ví dụ 2 mà có vẻ dường như ta đã biết, ta sẽ nhân vào đó con số $t$.


Ví dụ 1: Giải hệ phương trình: $\left\{\begin{matrix} x^2+y^2=\frac{1}{5} & \\4x^2+3x-\frac{57}{25}=-y(3x+1) & \end{matrix}\right.$

Nhân $t$ vào phương trình đầu rồi cộng theo vế với phương trình sau ta có:
$$ty^2+y(3x+1)+(t+4)x^2+3x-\frac{5t+57}{25}=0$$
Xem đây là phương trình bậc 2 ẩn $y$, xét:
\[{\Delta _y} = {(3x + 1)^2} - 4t\left[ {(t + 4){x^2} + 3x - \frac{{5t + 57}}{{25}}} \right]\]
\[ = (9 - 4{t^2} - 16t){x^2} + 6x(1 - 2t) + 1 + \frac{{4t(5t + 57)}}{{25}}\]
Để xuất hiện nhân tử như trên thì $\Delta _{y}=f^2(x)$ và như vậy thì:
$$(9-4t^2-16t)x^2+6x(1-2t)+1+\frac{4t(5t+57)}{25}=f^2(x)$$
$$\Leftrightarrow \Delta '_{x}=0\Leftrightarrow 9(1-2t)^2-4(9-16t-4t^2)\left [ 1+\frac{4t(5t+57)}{25} \right ]=0$$
$$\Leftrightarrow (1-2t)\left [ 1-2t-4(9+2t)\left [ 1+\frac{4t(5t+57)}{25} \right ] \right ]=0$$
Dễ thấy $t=\frac{1}{2}$ là giá trị thỏa mãn.

$ \bullet $ Để có lời giải gọn và đẹp thì khi trình bày bài giải, chúng ta nhân thêm $2$ vào phương trình sau thay vì nhân $\frac{1}{2}$ vào phương trình đầu. Từ đó ta có lời giải gọn và đẹp như trên.


Xem lại ví dụ 2: Giải hệ phương trình: $\left\{\begin{matrix} (x-y)^2+y=3 & \\x^2+2xy-5y^2-5x+13y=6 & \end{matrix}\right.$

Chúng ta cũng thực hiện công việc nhân $t$ như trên: Nhân $t$ vào phương trình đầu rồi cộng theo vế 2 phương trình ta được:
$$(t-5)y^2+y\left [ 2x(t-1)+t+13 \right ]+(t+1)x^2-5x-3(t+2)$$
$$\Delta _{y}=\left [ 2x(t-1)+t+13 \right ]^2-4(t-5)\left [ (t+1)x^2-5x-3(t+2) \right ]$$
$$=8(3+t)x^2-4x(t^2+7t+12)+9t^2+2t+249=f^2(x)$$
Khi xem xét phương trình này thì nhận thấy ngay $t=-3$ sẽ cho ta $f^2(x)=18^2$ vì để ý $t^2+7t+12=(t+3)(t+4)$. Từ đó ta có lời giải.

Một số ví dụ thêm:

Ví dụ 3: Giải hệ phương trình: $\left\{\begin{matrix} x^2+y^2=xy+x+y & \\x^2-y^2=3 & \end{matrix}\right.$

Ví dụ 4: Giải hệ phương trình: $\left\{\begin{matrix} x^2+2y^2=3x-2 & \\2(x+y-1)=2xy & \end{matrix}\right.$

Ví dụ 5: (THTT số 379, tháng 1 năm 2011)
Giải hệ phương trình: $\left\{\begin{matrix} y^2=(5x+4)(4-x) & \\y^2-5x^2-4xy+16xy-8y+16=0 & \end{matrix}\right.$

Ví dụ 6: (ĐH CĐ khối A, năm 2008)
Giải hệ phương trình: $\left\{\begin{matrix} x^2+y+x^3y+xy^2+xy=-\frac{5}{4} & \\x^4+y^2+xy(1+2x)=-\frac{5}{4} & \end{matrix}\right.$

Ví dụ 7: Giải hệ phương trình: $\left\{\begin{matrix} x^2-y^2=3 & \\x^2+y^2=xy+x+y & \end{matrix}\right.$

$ \bullet $ Một số lưu ý khi sử dụng phương pháp (Xem ở phần tản mạn)

Cần linh hoạt trong việc chọn lựa nhân $t$ ở phương trình nào để thuận lợi trong việc phân tích.
……


$ \bullet $ Mở rộng phương pháp:

$ \bullet $ Cở sở suy luận: bạn có nghĩ, liệu có bắt buộc bậc của $x$ và $y$ trong hệ phải là bậc 2 cả. Đúng, để luôn giải được thì nhất thiết phải yêu cầu là cả 2 đều bậc 2. Tuy nhiên, cái hay của Toán chính là đa dạng, muôn màu muôn vẻ, bắt buộc chúng ta phải luôn tinh tế, sáng tạo hơn nữa trong phương pháp và suy nghĩ. Biết 1 chưa chắc đã giải được 10. Trước hết, hãy xem cái hệ sau.

Ví dụ 8: (Thi thử ĐH CĐ năm 2011)
Giải hệ phương trình: $\left\{\begin{matrix} x^4+2x^3y+x^2y^2=2x+9 & \\x^2+2xy=6x+6 & \end{matrix}\right.$

Bậc cao nhất của $x$ là 4, nhưng bậc của $y$ lại là 2. Hơn thế nữa, nếu quan sát tinh ý hơn:
$$\left\{\begin{matrix} x^4+2x^2(xy)+(xy)^2=2x+9 & \\x^2+2xy=6x+6 & \end{matrix}\right.$$
(thì nên gom và xem $xy$ là ẩn)

Nhân thêm hằng số $t$ vào phương trình sau rồi cộng theo vế với phương trình đầu, ta được:
$$x^2y^2+2xy(t+x^2)+x^4+tx^2-2x(1+3t)-9-6t=0$$
$$\Delta '_{xy}=(t+x^2)^2-x^4-tx^2+2x(1+3t)+9+6t=tx^2+2x(1+3t)+(t+3)^2=f^2(x)$$
$$\Leftrightarrow \Delta '_{x}=(1+3t)^2-4t(t+3)^2=0$$
Dễ thấy ngay $t=1$ là một nghiệm của phương trình nên hệ số cần nhân chính là 1.

Việc trình bày lời giải còn lại xin dành cho bạn đọc.

$ \bullet $ Hệ này khá đặc biệt nhưng vì rút gọn ta thu được $\Delta _{xy}$ là một tam thức bậc 2. Qua đó cơ sở phương pháp của chúng ta vẫn áp dụng được. Nhưng. Ví dụ sau thì sao?

Ví dụ 9: Giải hệ phương trình: $\left\{\begin{matrix} 1+x^2y^2=19x^2 & \\xy^2+y=-6x^2 & \end{matrix}\right.$

$ \bullet $ Nhận xét: chú ý bậc cao nhất của $y$ như trên vẫn là bậc 2. Nhưng có vấn đề gì cần bàn ở đây?
Nháp: Nhân $t$ vào phương trình sau rồi cộng lại.


Còn tiếp ...

Mời bạn cùng thảo luận tại: http://diendantoanho...ệ-phương-trinh/



#340082 Vài bài toán hay về Bất đẳng thức lượng giác trong tam giác (phần 1)

Gửi bởi Ban Biên Tập trong 25-07-2012 - 16:27


VÀI BÀI TOÁN HAY VỀ ĐẲNG THỨC LƯỢNG TRONG TAM GIÁC

Đoàn Quốc Khánh, học sinh Lớp 11A1


trường THPT Ngọc Lặc, huyện Ngọc Lặc, tỉnh Thanh Hóa



I. Lời giới thiệu

Trong môn hình học ở trường phổ thông, hình học phẳng có khá nhiều phân môn, thể loại, và hình tam giác, có vai trò rất đặc biệt. Việc chứng minh nhiều định lý và giải rất nhiều bài toán hình học đòi hỏi phải vận dụng hợp lý nhiều kiến thức về hình tam giác(tam giác bằng nhau, tam giác đồng dạng, các đường thẳng đặc biệt trong tam giác, v.v…)
Hình tam giác đã được nhiều nhà toán học trên thế giới nghiên cứu từ hàng nghìn năm nay và mãi cho đến những năm gần đây, nhiều tính chất, định lý mới, hoặc nhiều cách chứng minh mới của các định lý đã biết lần lượt ra đời. Ở bài viết này, tác giả xin giới thiệu đến bạn đọc những định lý, những bài toán hay về đẳng thức lượng giác trong tam giác, bao gồm định lý Stewart, định lý Morley, định lý Steiner-Lenmus về tam giác cân, bài toán Napoleon … và những mở rộng, chú ý, cách chứng minh độc đáo của nhiều nhà toán học cũng được nêu ra trong bài viết này, chúng ta hãy cùng tìm hiểu.



Hình đã gửi


Hình đã gửi


II. Định lý STEWART


Bài toán: Cho$\Delta ABC$. $D$ là một điểm trên cạnh $BC$. Đặt $AD = d, BD = m, DC = n$. Khi đó ta có công thức sau: $a{d^2} = m{b^2} + n{c^2} - amn$
Lời giải.


Hình đã gửi



Kẻ đường cao $AH$. Xét hai tam giác $ABD$ và $ACD$ và theo định lý hàm số $cos$, ta có:
\[{c^2} = {d^2} + {m^2} - 2md\cos \widehat {ADB} = {d^2} + {m^2} - 2m.HD\,\,\,\,\,\,\,(1)\]
$${b^2} = {d^2} + {n^2} - 2dn\cos \left( {\pi - \widehat {ADB}} \right) = {d^2} + {n^2} + 2dn\cos \widehat {ADB} = $$
$$={d^2} + {n^2} + 2nHD\,\,\,\,\,\,(2)$$
Nhân từng vế $(1), (2)$ theo thứ tự với $n$ và $m$ rồi cộng lại, ta có:
$$n{c^2} + m{b^2} = {d^2}(n + m) + mn(m + n) \,\,\,\,\,\,\,(3)$$
Do $m + n = a$, nên từ $(3)$ ta có: $a{d^2} = m{b^2} + n{c^2} - amn$

Định lý Stewart chứng minh xong.
Chú ý:
$ \bullet $ Stewart (1717 – 1785) là nhà toán học và thiên văn học người Scotland.
$ \bullet $ Nếu trong hệ thức Stewart xét $AD$ là đường trung tuyến, thì từ hệ thức Stewart có:
\[am_a^2 = \frac{a}{2}{b^2} + \frac{a}{2}{c^2} - a\frac{a}{2}\frac{a}{2} \Leftrightarrow m_a^2 = \frac{{2{b^2} + 2{c^2} - {a^2}}}{4}\,\,\,\,\,\,\,\,\,\,(4)\]
Hệ thức trên chính là hệ thức xác định trung tuyến quen biết trong tam giác.
$ \bullet $ Nếu trong hệ thức Stewart xét $AD$ là phân giác. Khi đó theo tính chất đường phân giác trong ta có:
\[\frac{m}{c} = \frac{n}{b} \Leftrightarrow m = \frac{{ac}}{{b + c}}\,\,\,\,\,\,\text{và}\,\,\,\,\,\,n = \frac{{ab}}{{b + c}}\]
Từ hệ thức Stewart có:
\[al_a^2 = \frac{{ac}}{{b + c}}{b^2} + \frac{{ab}}{{b + c}}{c^2} - a\frac{{{a^2}bc}}{{{{\left( {b + c} \right)}^2}}} \Rightarrow l_a^2 = \frac{{bc\left[ {{{\left( {b + c} \right)}^2} - {a^2}} \right]}}{{{{\left( {b + c} \right)}^2}}}\,\,\,\,\,\,\,\,\,(5)\]
Chú ý rằng: $$2{\cos ^2}\frac{A}{2} = 1 + \cos A = 1 + \frac{{{b^2} + {c^2} - {a^2}}}{{2bc}} = \frac{{{{(b + c)}^2} - {a^2}}}{{2bc}}\,\,\,\,\,\,\,\,\,\,(6)$$
Từ $(5)$ và $(6)$ suy ra:
$$l_a^2 = \frac{{4{b^2}{{\cos }^2}\frac{A}{2}}}{{{{(b + c)}^2}}} \Rightarrow {l_a} = \frac{{2bc\cos \frac{A}{2}}}{{b + c}}$$
Hệ thức trên chính là hệ thức xác định đường phân giác.
Vậy hệ thức Stewart là tổng quát hoá của các hệ thức xác định đường trung tuyến và đường phân giác đã quen biết.
III. Định lý MORLEY
Bài toán: Cho $\Delta ABC$. Ở mỗi góc của tam giác vẽ hai đường chia góc đó ra làm ba phần bằng nhau. Các đường ấy cắt nhau tại $X, Y, Z$ (hình vẽ). Chứng minh rằng $\Delta XYZ$ đều.
Lời giải.
Hình đã gửi


Đặt $A = 3\alpha $, $B = 3\beta $,$C = 3\gamma $. Gọi các cạnh $BC, CA, AB$ và đường kính đường tròn ngoại tiếp $\Delta ABC$ là $a, b, c, d$. Theo định lý hàm số $sin$ trong$\Delta CYA$, ta có:
$$\frac{{CY}}{{\sin \alpha }} = \frac{b}{{\sin \left( {{{180}^o} - \alpha - \gamma } \right)}}\,\,\,\,\,\,\,\,\,\,\,(1)$$
Do $\alpha + \gamma = {60^o} - \beta $, vậy từ $(1)$ suy ra
\[{CY = b\frac{{\sin \alpha }}{{\sin \left( {{{120}^o} + \beta } \right)}} = d\sin 3\beta \frac{{\sin \alpha }}{{\sin \left( {{{60}^o} - \beta } \right)}}\,\,\,\,\,\,\,\,\,(2)}\]
Ta có: $$\sin 3\beta = 3\sin \beta - 4{\sin ^3}\beta = 4\sin \beta \left[ {{{\left( {\frac{{\sqrt 3 }}{2}} \right)}^2} - {{\sin }^2}\beta } \right] =$$
$$=4\sin \beta \left( {{{\sin }^2}{{60}^o} - {{\sin }^2}\beta } \right) = 4\sin \beta \sin \left( {{{60}^o} + \beta } \right)\sin \left( {{{60}^o} - \beta } \right)\,\,\,\,\,\,\,\,\,\,\,(3)$$
Thay $(3)$ vào $(2)$ có: $n = CY = 4d\sin \alpha \sin \beta \sin \left( {{{60}^o} + \beta } \right)$

Lí luận tương tự có: $m = CX = 4d\sin \alpha \sin \beta \sin \left( {{{60}^o} + \alpha } \right)$
Trong $\Delta CXY$, áp dụng định lý hàm số $cos$, ta có:
\[X{Y^2} = {m^2} + {n^2} - 2mn\cos \gamma = \]
\[ = 16{d^2}{\sin ^2}\alpha {\sin ^2}\beta \left[ {{{\sin }^2}\left( {{{60}^o} + \alpha } \right) + {{\sin }^2}\left( {{{60}^o} + \beta } \right) - 2\sin \left( {{{60}^o} + \alpha } \right)\sin \left( {{{60}^o} + \beta } \right)\cos \gamma } \right]\,\,\,\,\,\,\,\,(4)\]
Do $\left( {{{60}^o} + \alpha } \right) + \left( {{{60}^o} + \beta } \right) + \gamma = {180^0}$, nên xét $\Delta EFG\,\,\,\,\,\text{với}\,\,\,\,\widehat E = {60^o} + \alpha ,\widehat F = {60^o} + \beta ,\widehat G = \gamma .$
Gọi $d_1$ là đường kính đường tròn ngoại tiếp tam giác này. Theo định lý hàm số $cos$ trong tam giác này có:
\[{FG = e = {d_1}\sin \left( {{{60}^o} + \alpha } \right) \Rightarrow \sin \left( {{{60}^o} + \alpha } \right) = \frac{e}{{{d_1}}}}\]
\[{EG = f = {d_1}\sin \left( {{{60}^o} + \beta } \right) \Rightarrow \sin \left( {{{60}^o} + \beta } \right) = \frac{f}{{{d_1}}}}\]
\[{EF = g = {d_1}\sin \gamma \Rightarrow \sin \gamma = \frac{g}{{{d_1}}}}\]
Vậy thay vào $(4)$, ta có:
\[{X{Y^2} = 16{d^2}{{\sin }^2}\alpha {{\sin }^2}\beta \frac{{{e^2} + {f^2} - 2ef\cos \gamma }}{{d_1^2}}}\]
\[{ = 16{d^2}{{\sin }^2}\alpha {{\sin }^2}\beta \frac{{{g^2}}}{{d_1^2}} = 16{d^2}{{\sin }^2}\alpha {{\sin }^2}\beta {{\sin }^2}\gamma }\]
\[{ \Rightarrow XY = 4d\sin \alpha \sin \beta \sin \gamma }\]
Do vai trò bình đẳng, ta cũng có \[XZ = ZY = 4d\sin \alpha \sin \beta \sin \gamma \Rightarrow XY = YZ = ZX.\]
Vậy $\Delta XYZ$ là tam giác đều (đpcm).
Chú ý:
$ \bullet $ Frank Morley (1860 – 1937) sinh tại Anh, nhưng hầu như suốt đời sống ở Mĩ. Trong vài chục năm ông là giáo sư toán học ở trường đại học tổng hợp thuộc bang Baltimore. Bản thân học cách chứng minh của ông rất phức tạp. Cách chứng minh ở trên là của nhà toán học Ấn Độ Naranengar tìm ra vào năm 1909. Một nhà toán học Ấn Độ khác là Xachianarian cho cách giải "phi lượng giác" (chỉ dùng đến kiến thức hình học lớp 9)
$ \bullet $ Định lý về đường chia ba góc được Morley tìm ra từ 1899, nhưng mãi đến năm 1914 ông mới công bố cách chứng minh và mở rộng định lý với việc xét không chỉ các đường chia ba góc trong mà cả các đường chia ba góc ngoài của tam giác. Định lý Morley đã hấp dẫn nhiều người, trong đó có nhà toán học Pháp nổi tiếng Henri Lebesgue (1875 – 1941). Năm 1939, Lebesgue công bố chứng minh sơ cấp của định lý này. Ông xét các đường chia ba các góc trong và ngoài của tam giác (có tất cả 12 đường), và đã chứng minh được rằng trong các giao điểm của các đường đó có 27 bộ ba điểm là các đỉnh của tam giác đều.



Còn nữa ...




#335135 Chuyên đề Hệ phương trình

Gửi bởi Ban Biên Tập trong 13-07-2012 - 09:20

CHUYÊN ĐỀ HỆ PHƯƠNG TRÌNH

Phạm Hùng Vương

Học sinh lớp 12C1 trường THPT Phan Đăng Lưu, Nghệ An


I. Lời nói đầu.

Chuyên đề là kết quả thu được qua một thời gian học tập và nghiên cứu của bản thân về hệ phương trình. Tuy nhiên có thể nói rằng, đó là sự kết tinh qua nhiều thế hệ, là sự giúp đỡ, là sự học hỏi từ những người bạn của mình cũng như rất nhiều yếu tố khác.

Để đạt hiệu quả cao khi tham khảo chuyên đề này, xin được trích dẫn mấy lời của nhà giáo G.Polya:
" [...] Một số bài toán có nêu lời giải đầy đủ (tuy vắn tắt), đối với một số bài khác, chỉ vạch ra mấy bước giải đầu tiên, và đôi khi chỉ đưa ra kết quả cuối cùng.

Một số bài toán có kèm thêm chỉ dẫn để giúp người đọc giải được dễ dàng hơn. Chỉ dẫn cũng có thể nằm trong những bài toán khác ở gần bài toán đang xét. Nên đặc biệt lưu ý đến những nhận xét mở đầu trước từng bài tập hay cả một nhóm bài tập gặp thấy trong chương.

Nếu chịu khó, gắng sức giải một bài toán nào đó thì dù không giải nổi đi chăng nữa, bạn đọc cũng thu hoạch được nhiều điều bổ ích. Chẳng hạn, bạn đọc có thể giở ra xem (ở cuốn sách) phần đầu mỗi lời giải, đem đối chiếu với những suy nghĩ của bản thân mình, rồi gấp sách lại và thử gắng tự lực tìm ra phần còn lại của lời giải.

Có lẽ thời gian tốt nhất để suy nghĩ, nghiền ngẫm về phương pháp giải bài toán là lúc bạn vừa tự lực giải xong bài toán hay vừa đọc xong lời giải bài toán trong sách, hay đọc xong phần trình bày phương pháp giải trong sách. Khi vừa hoàn thành xong nhiệm vụ, và các ấn tượng hãy còn "nóng hổi", nhìn lại những nổ lực vừa qua của mình, bạn đọc có thể phân tích sâu sắc tính chất của những khó khăn đã vượt qua. Bạn đọc đọc có thể tự đặt cho mình nhiều câu hỏi bổ ích: "Khâu nào trong quá trình giải là quan trọng nhất? Khó khăn chủ yếu là ở chỗ nào? Ta có thể làm gì cho tốt hơn? Chi tiết ấy mình cũng đã liếc qua mà không chú ý đến - muốn "nhìn thấy" chi tiết này thì đầu óc phải có tư chất ra sao? Liệu ở đây có một cách gì đó đáng lưu ý để sau này gặp một tình huống tương tự, ta có thể áp dụng được không?" Tất cả những câu hỏi đó đều hay cả, và cũng còn nhiều câu hỏi bổ ích khác nữa, nhưng câu hỏi hay nhất chính là câu hỏi tự nhiên nảy ra trong óc, không cần ai gợi ý cả!"




(trích "Mấy lời khuyên và chỉ dẫn" -G.Polya trong "Sáng tạo toán học")


Do thời gian cũng như 1 số vấn đề khác như kiến thức, trình bày,.. mà chuyên đề này còn khá nhiều khiếm khuyết. Rất mong được các bạn quan tâm và chia sẻ đề hoàn thiện chuyên đề hơn. Hi vọng nó sẽ là tài liệu bổ ích giúp chúng ta vượt qua 1 chẳng nhỏ trong chặng đường chinh phục toán học.

II. MỘT SỐ PHƯƠNG PHÁP CŨ.

1. Hệ phương trình đối xứng kiểu I.

Nhận dạng: Hệ đối xứng kiểu I: gồm 2 phương trình ẩn x,y mà vai trò x,y trong mỗi phương trình là như nhau. Ví dụ: $\left\{\begin{matrix} a(x+y)+bxy=c\\x^2+y^2=c \end{matrix}\right.$. Và phương pháp giải là đặt ẩn phụ: $S = x + y, P = xy$. Giải tìm $S, P$ sau đó sử dụng định lí Vi-et, dễ thấy $x,y$ là nghiệm của phương trình: $X^2 - S.X + P = 0$
Cùng xem xét 1 vài ví dụ (cách giải và một số hướng giải quyết mới)

Ví dụ 1: (Đề thi HSG lớp 9 Tỉnh Bến Tre năm 2009-2010)
Giải hệ phương trình: $\left\{\begin{matrix}x^2+y^2-2x-2y=6 \\x+y-xy=5 \end{matrix}\right.$

Bài giải: Đặt $S = x + y, P = xy$, ta thu được hệ mới tương đương:
$$\left\{\begin{matrix} S^2-2P-2S=6\\S-P=5 \end{matrix}\right. \Leftrightarrow \left\{\begin{matrix} S^2-4S+4=0\\ P=S-2 \end{matrix}\right.\Leftrightarrow \left\{\begin{matrix} S=2\\P=-3 \end{matrix}\right.$$
Như vậy, theo định lí Vi-ét, $x,y$ là nghiệm của phương trình:
$${X^2} - 2X - 3 = 0 \Leftrightarrow (X - 3)(X + 1) = 0 \Rightarrow \left[ \begin{array}{l}
x = 3,y = - 1\\
x = - 1,y = 3
\end{array} \right.$$
Vậy hệ có 2 nghiệm $(x;y)$ thỏa mãn là: $(-1;3)$ và $(3;-1)$.

Những bài như thế này và bài giải như vậy đã trở nên quen thuộc, không còn mới lạ. Tuy nhiện, cũng có 1 số bài hệ, dù biết là đối xứng kiểu I, nhưng lại phải làm gì để sử dụng được? Hãy xem ví dụ:

Ví dụ 2: (ĐH-CĐ Khối A năm 2006)
Giải hệ phương trình: $\left\{ \begin{matrix} x+y-\sqrt{xy}=3\\\sqrt{x+1}+\sqrt{y+1}=4 \end{matrix}\right. $

Bài giải:

Ý tưởng 1: Thử đặt như cũ: $S=x+y, P=xy$, hệ khi đó trở thành:
$$\left\{ \begin{matrix} x+y-\sqrt{xy}=3\\ x+y+2+\sqrt{xy+x+y+1}=16 \end{matrix}\right. \Leftrightarrow \left\{ \begin{matrix} S-\sqrt{P} =3\\ S+2\sqrt{P+S+1}=14 \end{matrix}\right.$$
$$\Leftrightarrow \left\{ \begin{matrix} S=\sqrt{P}+3\\2\sqrt{P+\sqrt{P}+4}=11-\sqrt{P} \end{matrix}\right. \Leftrightarrow \left\{ \begin{matrix} S=\sqrt{P}+3\\3P-26\sqrt{P}-105=0 \\ 0 \leq P \leq 121 \end{matrix}\right. $$
Đến đây, giải tìm $P$, sau đó quay lại giải tìm ra nghiệm $x,y$. ( chú ý điều kiện)

Hơn nữa, luôn nhớ: $S^2 \geq 4P$ để loại bớt nghiệm.

Ý tưởng 2: Đặt ẩn $a =\sqrt{x +1},b =\sqrt{y+1}$ nhằm làm đơn giản 1 phương trình của hệ. (kĩ thuật đặt ẩn làm gọn này rát có ý nghĩa, đặc biệt trong bất đẳng thức (BĐT) có giả thiết rườm rà, với phương trình hay hệ cũng vậy). Khi đó:
$$HPT \Leftrightarrow \left\{ \begin{matrix} a+b=4\\a^2+b^2-2-\sqrt{(a^2-1)(b^2-1)}=3 \end{matrix}\right. \Leftrightarrow \left\{ \begin{matrix} S=4\\S^2-2P-2-\sqrt{P^2-S^2+2P+1}=3 \end{matrix}\right. \Leftrightarrow \left\{ \begin{matrix} S=4\\\sqrt{P^2+2P-15}=11-2P \end{matrix}\right. $$
Trong đó $S=a+b, P=ab$. Đến đây, ta cũng có thể giải tương tự.

Ví dụ 3: (Thi thử ĐH-CĐ, THPT chuyên Nguyễn Huệ 2011)
Giải hệ phương trình: $\left\{ \begin{matrix} \sqrt{x+1}+\sqrt{y-1}=4\\\sqrt{x+6}+\sqrt{y+4}=6 \end{matrix}\right.$

Ví dụ 4:(Đề thi HSG lớp 9 tỉnh Nghệ An năm 2009-2010)
Giải hệ phương trình: $\left\{ \begin{matrix} \frac{1}{x}+\frac{1}{y}+\frac{1}{z}=2\\ \frac{2}{xy}-\frac{1}{z^2}=4 \end{matrix}\right.$

Ví dụ 5: Giải hệ phương trình: $\left\{ \begin{matrix} (x+y)(1+xy)=4xy\\(x^2+y^2)(1+x^2y^2)=4x^2y^2 \end{matrix}\right.$

Thực ra, dạng hệ đối xứng kiểu I có hướng giải khá đơn giản, rõ ràng với việc đặt ẩn và sử dụng định lí Vi-ét. Chính vì vậy mà hệ đối xứng kiểu I thường gắn với việc giải và biện luận, một sở trường của phương pháp này! Chúng ta cùng xét một số ví dụ sau.

Ví dụ 6: (Đề thi HSG lớp 9 tỉnh Hà Nội năm 2009-2010)
Tìm $a$ để hệ phương trình sau có nghiệm duy nhất: $\left\{ \begin{matrix} ay+x+y=a+1\\x^2y+xy^2=a \end{matrix}\right.$

Bài giải: Đặt : $S = x + y, P = xy$, ta có hệ mới: $\left\{ \begin{matrix} S+P=a+1\\SP=a \end{matrix}\right.$

Theo Vi-ét, $S$ và $P$ là nghiệm của phương trình: $X^2-(a +1)X + a = 0 (1)$

Hơn nữa, cũng theo Vi-ét $x,y$ lại là nghiệm của phương trình: $X^2 -S.X + P = 0 (2).$

Do đó, để hệ có 1 nghiệm duy nhất thì $(2)$ có nghiệm duy nhất, tức $\Delta _{(2)} = 0 \Leftrightarrow S^2 = 4P \Leftrightarrow x = y$ Hoặc có thể dùng nhận xét: do vai trò $x, y$ trong mỗi phương trình của hệ là như nhau nên nếu hệ có nghiệm $(m; n)$ thì nó cũng có nghiệm $(n; m)$. Như vậy để hệ có nghiệm duy nhất thì cần có $x = y$. Thế vào được:
$$\left\{ \begin{matrix} x^2+2x=a+1\\2x^3=a \end{matrix}\right. \Leftrightarrow \left\{ \begin{matrix} x^2+2x-(a+1)=0 \,\,\,\,\,\,(*)\\x=\sqrt[3]{\frac{a}{2}} \end{matrix}\right. $$
Để hệ có nghiệm duy nhất thì $(*)$ có duy nhất 1 nghiệm $x=\frac{-2}{2.1}=-1\Rightarrow \sqrt[3]{\frac{a}{2}}=-1\Leftrightarrow a=-2.$

Thử lại thấy thỏa mãn. Kết luân giá trị cần tìm là $a=-2.$

Ví dụ 7: (Đề thi HSG lớp 9 Tỉnh Hưng Yên năm 2009-2010)
Tìm $m$ để hệ phương trình sau có nghiệm: $\left\{ \begin{matrix} \sqrt{x}+\sqrt{y}=m\\x+y-\sqrt{xy}=m \end{matrix}\right.$

Ví dụ 8: (Thi thử ĐH-CĐ THPT Lương Ngọc Quyến, Thái Nguyên 2011)

Tìm $a$ để hệ phương trình sau có nghiệm: $\left\{ \begin{matrix} \sqrt{x+1}+\sqrt{y+1}=a\\x+y=2a+1 \end{matrix}\right.$

Nếu đơn thuần chỉ là hệ đối xứng kiểu I thì chắc chắn nó sẽ nhanh chóng được chúng ta giải quyết. Chính vì vậy, mà sau đây sẽ các ví dụ cần dùng các kĩ thuật nhỏ chuyển về hệ đối xứng kiểu I. (Phần kĩ năng sẽ trình bày rõ hơn ở mục sau).

Ví dụ 9: Giải hệ phương trình: $\left\{ \begin{matrix} (x-1)^2+6(x-1)y+4y^2=20\\x^2+(2y+1)^2=2 \end{matrix}\right.$

Nhận xét: Quan sát thì thấy ngay không thể là hệ đối xứng kiểu I. Nhưng! Hãy xem hướng giải sau:

Bài giải: Đặt $a = x -1, b = 2y$ thì hệ trở thành: $\left\{ \begin{matrix} a^2+3ab+b^2=20\\(a+1)^2+(b+1)^2=2 \end{matrix}\right.$. Đúng là hệ đối xứng kiểu I!

Bây giờ, thì có thể đi tiếp theo phương pháp được rồi.

Có thể nói rằng, vấn đề đặt gọn luôn ẩn hiện 1 điều gì đó rất thú vị nếu ta tinh ý trong các biểu thức nhìn có vẻ có vấn đề. Như ví dụ trên chẳng hạn, phải biết nghi ngờ $(x -1)$, khi nó được đặt trong ngoặc.

Hãy tiếp tục với hệ sau:

Ví dụ 10: Giải hệ phương trình: $\left\{ \begin{matrix} x^2+6xy+4y^2=19+2y+6y \,\,\,\,\,\,(1) \\ x^2+4y^2=1-4y \,\,\,\,\,\,\,\, (2) \end{matrix}\right.$

Có thể thấy, cả 2 ví dụ 10 và ví dụ 11 đều chỉ là một. Nhưng nếu nghiệm theo cách đặt ẩn gọn thì đặt cái nào.

Nếu đặt $a = x-1,\,\,b = 2y$ như trên thì tại sao lại biết mà đặt như vậy. Đây chính là vấn đề cần bàn. Nếu đi theo phân tích phương trình $(1)$ thì sẽ có khá nhiều phương án: chẳng hạn nghĩ đến hằng đẳng thức: $(1) \Leftrightarrow (x + 3y)^2 -5y2 = 19 + 2(x + 3y), ..v.v$. Có khá nhiều đẳng thức có thể nghĩ tới để đặt.

Nhưng với phương trình $(2)$ thì lại khác: nó chỉ có một đằng thức cần chú ý: $(2) \Leftrightarrow x^2 + (2y +1)^2 = 2.$ Như vậy, ý tưởng đặt làm gọn $(2)$ mở ra: $a = 2 y +1$, hơn nữa có thể thấy ở phương trình $(1)$ hệ số của $y$ luôn chẵn, khi thế có thể thế $2y = a -1$ (đây không phải là một trùng hợp ngẫu nhiên. Hãy nghĩ vậy).

Việc làm còn lại thì khá rõ rồi, ta cũng thu được một hệ đối xứng kiểu I và tiếp tục giải.

Hãy thử với các ví dụ:

Ví dụ 11: Giải hệ phương trình: $\left\{ \begin{matrix} x^4-4x^2+(y-3)^2=0\\x^2y+x^2+2y-22=0 \end{matrix}\right.$

Ví dụ 12: Giải hệ phương trình: $\left\{ \begin{matrix} (x-y)^2=1-x^2y^2\\x(xy+y+1)=y(xy+1)+1 \end{matrix}\right.$

Thậm chí còn có những bài có những cách đặt đưa về hệ đối xứng rất thú vị, khó mà thấy được nếu không qua chút biến đổi. Vì vậy, hãy cố gắng quan sát và đặt gọn phù hợp.

Ví dụ 13: Giải hệ phương trình: $\left\{ \begin{matrix} x^2+y^2=5+4x-4y\\3x+xy-y=15 \end{matrix}\right.$

Ví dụ 14: (THTT số 379 năm 2009)
Giải hệ phương trình: $\left\{ \begin{matrix} xy-3x-2y=16\\x^2+y^2-2x-4y=33 \end{matrix}\right.$

Ví dụ 15: Giải hệ phương trình: $\left\{ \begin{array}{l}
{x^2} + {y^2} = 2\\
2{x^2} + 3xy - 2{y^2} + 3x + y = 7
\end{array} \right.$ (xem giải ở mục II. phương pháp 02)

Hơn nữa, dạng hệ đối xứng kiểu I này rất hay vận dụng một hằng đẳng thức (đang có xu hương lớn trong các đề thi thử): $\frac{1}{x^2}+y^2=(\frac{1}{x}+y)^2-2 \frac{y}{x}.$. Tiếp tục với các ví dụ sau, bạn sẽ thấy rõ.

Ví dụ 16: Giải hệ phương trình: $\left\{ \begin{matrix} \frac{y}{x^2}+\frac{y^2}{x}=6\\\frac{1}{x^2}+y^2=5 \end{matrix}\right.$

Ví dụ 17: Giải hệ phương trình: $\left\{ \begin{matrix} xy+y^2+x-7y=0\\xy+x^2-12y=0 \end{matrix}\right.$

Ví dụ 18: Giải hệ phương trình: $\left\{ \begin{matrix} (x^2+y^2)(1+\frac{1}{xy})^2=9\\(x^3+y^3)(1+\frac{1}{xy})^3 =4 \end{matrix}\right.$

Ví dụ 19: Giải hệ phương trình: $\left\{ \begin{matrix} xy(2x+y-6)+2x+y=0\\(x^2+y^2)(1+\frac{1}{xy})^2=8 \end{matrix}\right.$

Và cả một dạng (ở phần cuối chuyên đề)

2. Hệ phương trình đối xứng kiểu II.

Nhận dạng: Cũng như loại I, loại II cũng “đối xứng” nhưng là đối xứng giữa 2 phương trình chứ không không phải là đối xứng trong từng phương trình như kiểu I.

Một cách nhận dạng khác nữa là cho $x = y$ thì 2 phương trình của hệ như nhau. Hay nói cách khác $x = y$ chính là nghiệm của hệ. Đây chính là đặc điểm khai thác của hệ này.

Phương pháp: Thông thường, ta trừ theo vế ta thu được nghiệm $x = y$, và 1 số nghiệm khác. Sau đó thay lại tìm ra nghiệm $(x; y)$.

Cùng xem xét một số ví dụ đơn giản.
Ví dụ 20: Giải hệ phương trình: $\left\{ \begin{matrix} x^2+y=5x+3\\y^2+x=5y+3 \end{matrix}\right.$

Bài giải: Trừ theo của hệ thu đươc: $x^2 -y^2 = 6(x-y) \Leftrightarrow (x-y)(x + y-6) = 0$. Do đó, hệ phương trình đã cho tương đương với:

\[\left[ \begin{array}{l}
\left\{ \begin{array}{l}
x = y\\
{x^2} - 4x + 3 = 0
\end{array} \right.\\
\left\{ \begin{array}{l}
x + y = 6\\
{x^2} + x - 5\left( {6 - x} \right) = - 3
\end{array} \right.
\end{array} \right. \Leftrightarrow \left[ \begin{array}{l}
\left[ \begin{array}{l}
x = y = 1\\
x = y = 3
\end{array} \right.\\
\left[ \begin{array}{l}
x = 3,y = 3\\
x = - 9,y = 15
\end{array} \right.
\end{array} \right. \Leftrightarrow \left[ \begin{array}{l}
x = y = 1\\
x = y = 3\\
x = - 9,y = 15
\end{array} \right.\]
Vậy hệ phương trình đã cho có nghiệm là $\left( {x;y} \right) = \left\{ {\left( { - 9;15} \right),\left( {1;1} \right),\left( {3;3} \right)} \right\}$

Ví dụ 21: Giải hệ phương trình: $\left\{ \begin{matrix} x^3=2y+1\\y^3=2x+1 \end{matrix}\right.$

Bài giải: Trừ theo vế của hệ ta thu được: $x^3 - y^3 = 2 (y - x) \Leftrightarrow (x - y)(x^2 + xy + y^2 + 2) = 0 \Leftrightarrow x = y$

Vì $x^2 + xy + y^2 + 2 = (x+\frac{y}{2})^2+\frac{3}{4}y^2+2>0$. Như vậy thế $x = y$ vào hệ, ta chỉ cần giải phương trình:

\[{x^3} - 2x + 1 = 0 \Leftrightarrow \left( {x - 1} \right)\left( {{x^2} + x - 1} \right) = 0 \Leftrightarrow \left[ \begin{array}{l}
x = 1\\
{x^2} + x - 1 = 0
\end{array} \right. \Leftrightarrow \left[ \begin{array}{l}
x = 1\\
x = \frac{{ - 1 \pm \sqrt 5 }}{2}
\end{array} \right.\]

Vậy hệ phương trình đã cho có nghiệm là $x = \left\{ {\frac{{ - 1 \pm \sqrt 5 }}{2};1} \right\}$

$ \bullet $ Chú ý: Khi trừ theo vế, ta thu đươc: $x^3 + 2 x = y^3 + 2 y$. Nếu không dùng phân tích trên, ta có thể tính đạo hàm: $f (t) = t^3 + 2t$ có $f'(t) = 3t^2 + 2 > 0$ nên suy ra: $x = y .$

$ \bullet $ Nhận xét: Đơn giản chỉ là trừ vế theo vế, nhưng với những bài khác nhau lại cần thêm những kĩ thuật khai thác khác nhau để là xuất hiện $(x – y)$. Hãy xem:

Ví dụ 22: (Thử sức trước kì thi, THTT số 407, 2011)
Giải hệ phương trình: $\left\{\begin{matrix} \sqrt{x+10}+\sqrt{y-1}=11 & \\\sqrt{y+10}+\sqrt{x-1}=11 & \end{matrix}\right.$

Bài giải: Điều kiện các phân thức có nghĩa: $x,y\geq 1$. Chú ý $x=y=1$ không là nghiệm của hệ nên trừ theo vế 2 phương trình của hệ và nhân lượng liên hợp ta có:
\[\sqrt {x + 10} - \sqrt {y + 10} + \sqrt {y - 1} - \sqrt {x - 1} = 0\]
\[ \Leftrightarrow \left( {x - y} \right)\left( {\frac{1}{{\sqrt {x + 10} + \sqrt {y + 10} }} - \frac{1}{{\sqrt {x - 1} }} + \frac{1}{{\sqrt {y - 1} }}} \right) = 0 \Leftrightarrow x = y\]
(Vì do $\sqrt{x+10}+\sqrt{y+10}>\sqrt{x-1}+\sqrt{y-1}$ nên biểu thức còn lại vô nghiệm).

Thế $x = y$ vào ta dễ dàng giải phương trình của hệ.

Còn tiếp...




#332038 Toán học trong bầu cử

Gửi bởi Ban Biên Tập trong 05-07-2012 - 07:52

BBT không đăng lại phần 3 vì phần này tác giả trình bày quan điểm chính trị của cá nhân.
Hệ thống bầu cử 1 lần có chuyển phiếu (single transferable vote, viết tắt là STV) để bầu cử một ai hay một cái gì đó, khi mà có ít nhất 3 ứng cử viên, như sau:
* Mỗi lá phiếu ghi tên (hay đánh dấu) các ứng cử viên theo thứ tự lựa chọn của cử tri: dòng đầu tiên là người mà cử tri thích nhất, dòng thứ hai là người mà cử tri muốn bầu nếu người mà mình thích nhất không được bầu, và cứ thế.(Không nhất thiết phải ghi tên toàn bộ các ứng cử viên, nếu những người nào mà cử tri hoàn toàn không thích bầu thì không cần cho vào danh sách).
Hình đã gửi
* Thuật toán bầu như sau:
- Đầu tiên đếm số phiếu của các ứng cử viên theo dòng thứ nhất (số cử tri đặt ứng cử viên lên hàng đầu). Nếu có ai đạt trên 50% số phiếu thì thắng, và việc bầu cử kết thúc. Nếu không ai đạt 50%, thì loại đi người đạt ít phiếu nhất trong lần đếm đầu tiên này, và chuyển sang lần đếm thứ hai.
- Ở lần đếm thứ hai, gạch tên ứng cử viên đã bị loại ra khỏi các lá phiếu. Ví dụ, nếu ứng cử viên bị loại đứng hàng đầu ở một lá phiếu nào đó, thì bây giờ ứng cử viên đứng hàng thứ 2 ở lá phiếu đó được chuyển lên thành hàng đầu , ứng cử viên đứng hàng thứ 3 được chuyển lên thành hàng 2, và cứ thế. Sau khi gạch tên ứng cử viên đã bị loại như vậy, thì lặp lại quá trình đếm: nếu có ứng cử viên nào đứng ở hàng đầu ở trên 50% số phiếu thì được bầu, còn nếu không thì loại đi ứng cử viên có số phiếu hàng đầu ít nhất, rồi tiếp tục như trên.


Hình đã gửi
Hệ thống bầu cử 1 lần có chuyển phiếu trên, và các dạng tương tự của nó, xuất hiện từ thế kỷ 19, và ngày nay nó được dùng trong các cuộc bầu cử ở khá nhiều nơi trên thế giới, trong đó có Úc, Anh, Ấn Độ, Hồng Kông, v.v. Giải Oscar về điện anh cũng được bầu theo STV. Hệ thống STV còn được biết đến với các tên gọi khác như: Instant runoff voting, alternative vote.
Hệ thống STV có nhiều điểm ưu việt rõ rệt so với hệ thống “simple plurality” (không cần quá bán mà chỉ cần số phiếu nhiều hơn các ứng cử viên khác để được bầu ngay vòng đếm phiếu đầu tiên) hiện còn được dùng ở nhiều nơi, và hệ thống bầu cử 2 vòng ở Pháp. Hệ thống “simple plurality” quá là rởm rít trong trường hơp có nhiều ứng cử viên, không chấp làm gì. So với hệ thống bầu hai vòng ở Pháp, thì hệ thống STV có các ưu điểm sau:
* Cử tri chỉ cần đi bầu 1 vòng, thay vì 2 vòng. Tổ chức bầu 2 vòng tốn kém về thời gian (có khi mất thêm cả tháng) và tiền bạc (tính theo đơn vị trăm triệu USD) so với là chỉ 1 vòng.
* Trong trường hợp có nhiều ứng cử viên, thì việc chỉ chọn 2 người vào vòng 2 nhiều khi cũng éo le chẳng kém gì việc để người được nhất vòng 1 thắng ngay. Đây là điều đã xảy ra trong bầu cử tổng thống Pháp năm 2002. Nếu như Pháp dùng hệ thống STV đã không xảy ra sự éo le đó.
* Hệ thống STV khiến cho người ta bầu thật sự theo suy nghĩ của mình hơn là hệ thống 2 vòng. Ở Pháp, người ta phải kêu gọi “voter utile” vòng 1 (tức là không bầu cho người mình thực sự thích nhất, mà bầu cho người mình không thích lắm nhưng có nhiều khả năng trúng cử nhất trong số các ứng cử viên còn lại mà mình thấy tạm được) để tránh khỏi các tình huống éo le khi có nhiều ứng cử viên. Nhưng kiểu “voter utile” đó là một thứ phản dân chủ, khi các cử tri (hay các đảng phái) bỏ phiếu ngược lại ý nguyện thực sự của mình hòng thao túng kết quả bầu cử.
Hệ thống STV chưa phải là “hoàn hảo”. Nó không thỏa mãn một số tính chất quan trọng, trong đó có tính chất đơn điệu (monotonicity) sau: nếu 1 cử tri tăng thứ tự lựa chọn 1 ứng cử viên nào đó lên trong lá phiếu bầu của mình, thì điều đó không thể làm hại ứng cử viên đó, mà chỉ có thể hoặc không ảnh hưởng gì hoặc làm tốt lên cho ứng cử viên đó. Ví dụ đơn giản sau cho thấy, trong một số trường hợp nào đó dùng STV , có thể làm hại một ứng cử viên bằng cách nâng anh ta lên:
100 người bầu cho 3 ứng cử viên A,B,C, với kết quả các là phiếu như sau:

36 ABC


(tức là 36 người chọn A hàng đầu, sau đó đến chọn B, và xếp C là phương án tồi nhất)



34 BCA
30 CAB


Trong lần đếm phiếu đầu tiên thì C bị loại (chỉ có 30 phiếu, ít nhất). Lần đếm thứ hai thì A thắng (được 66 phiếu, trong khi B vẫn chỉ được 34 phiếu)
Nay giả sử có 5 người thay vì chọn BCA lại chọn thành ABC, tức là nâng A từ thứ ba lên thứ nhất trong các lá phiếu của họ, kết quả sẽ thành


41 ABC
29 BCA
30 CAB


Nếu có 5 người nâng A lên như vậy, thì B bị loại trong lần đếm đầu, và A thua trong lần đếm thứ 2, và C thắng chứ không phải A thắng !
Theo định lý Gibbard-Satterthwaite thì thực ra không có một hệ thống bầu cử dân chủ nào là có thể hoàn toàn tránh khỏi lũng đoạn, nên ví dụ trên có lẽ không đáng ngạc nhiên lắm. Tuy nhiên, có các công trình cho thấy, lũng đoạn bầu cử trong hệ thống STV là vấn đề “NP-hard”, tức là trên thực tế thì không đáng sợ lắm chuyện người ta không thật lòng khi bầu cử theo hệ thống STV, xem: http://www.cs.duke.e...09/stv_hard.pdf. Bởi vậy, hệ thống STV có thể coi là khá tốt để chống lại các trò “strategic voting”.
Trong trường hợp mà cuộc bầu cử có nhiều người chứ không chỉ 1 người được bầu (ví dụ như là bầu vào quốc hội), thì các thuật toán của các hệ thống bầu cử STV không những cho phép chuyển phiếu từ các ứng cử viên đã bị loại sang các ứng cử viên mà cử tri chọn lựa tiếp theo, mà nó còn có thể cho phép chuyển bớt phiếu từ các ứng cử viên đã được bầu mà thừa phiếu để được bầu sang các ứng cử viên “cùng phe” khác. Thuật toán chuyển phiếu thừa này cho phép hệ thống bầu cử STV gần đạt tính chất tỷ lệ thuận (proportional, tức là nếu đảng phái nào hay nhóm nào có tỷ lệ bao nhiêu % cử tri ủng hộ, thì cũng có tỷ lệ gần như vậy người được bầu) hơn hẳn so với hệ thống bầu cử quốc hội 2 vòng ở Pháp hiện tại.



#331702 Laurent Schwartz - Người bạn lớn của VN .

Gửi bởi Ban Biên Tập trong 04-07-2012 - 00:42

BBT: Đây là bài viết của GS Nguyễn Duy Tiến gửi cho Diễn đàn toán học Tưởng niệm 10 năm ngày mất GS LAURENT SCHWARTZ (4/7/2002-4/7/2012)

NGƯỜI VIỆT NAM CHÚNG TÔI MÃI MÃI BIẾT ƠN ÔNG, GIÁO SƯ LAURENT SCHWARTZ
(Tưởng niệm 10 năm ngày mất GS LAURENT SCHWARTZ
4/7/2002-4/7/2012)

Hình đã gửi

GS LAURENT SCHWARTZ (05/03/1915 - 04/07/2002)



Trong hồi ký của giáo sư Laurent Schwartz có câu “Les Vietnamiens ne m’ou- blient pas” (Người Việt Nam không quên tôi). Đúng thế, những người Việt Nam chúng tôi không bao giờ quên Ông, Giáo sư Laurent Schartz nhà toán học vĩ đại, người đã tới thăm và giảng bài Việt Nam 3 lần vào những năm tháng khó khăn nhất của dân tộc ta.

Lần thứ nhất vào cuối năm 1968 với tư cách là thành viên của Tòa án Betrand Russell "xử tội ác đế quốc Mỹ trong cuộc chiến tranh với Việt Nam". Ông được Thủ Tướng Phạm Văn Đồngtiếp 3 tiếng, và được nói chuyện với Hồ Chủ Tịch 1 tiếng (một trường hợp hiếm có). Vì thời gian có hạn, nên Ông chỉ có một buổi làm việc với giới toán học Việt Nam. Hôm đó Ông nêu một nhận xét trong phương trình đạo hàm riêng. Tôi nhớ là Ông nói "nhận xét đơn giản này mở ra cho một hưóng mới của toán học hiện đại". Tôi không hiểu gì.

Sau đó, Ông đã ủng hộ 10.000 quan cho Việt Nam khi bệnh viện Bạch Mai bị máy bay Mỹ ném bom.

Lần thứ hai vào mùa xuân 1976, Ông giảng (trong 3 tuần liền (1), giáo sư Phan Đức Chính dịch) về ánh xạ Radon hóa (Radonifying maps) và một ít về Hình Học các không gian Banach (Geometry of Banach Spaces). Tôi thực sự may mắn, vì

1) Sau khi bảo vệ tiến sĩ (26/04/1974), trước thời hạn 18 tháng, với luận án "Một số vấn đã về xác suất trong không gian Banach", tôi được tiêp tục ở lại Tbilisi làm việc với nhóm nghiên cứu của giáo sư N. N. Vakhania (thầy giáo hưng dẫn luận án tiến sĩ của tôi). Có thể nói, Tbilisi là trung tâm toán học nổi tiếng toàn thế giới về hàm phức và cơ học với những nhà toán học lừng danh như Mushkhelisvily, Vekua (thầy của giáo sư Ngô Văn Lực), Bissadze (thầy của giáo sư Nguyễn Thừa Hợp). Nhưng vào những năm 60, ở Tbilisi xác suất và thống kê thì còn yếu. Nhóm nghiên cứu của giáo sư Vakhania là nhóm mới nổi lên nhờ có quan hệ tốt với trường phái xác suất của Mockva, đc biệt là được sự hỗ trợ của Yu. V. Prokhorov, V.V Sazonov. Lúc tôi tới Tbilisi (5/10/1971), Vakhania mới bảo vệ tiến sĩ khoa học được 2 năm, và S. Chobanyan mới bảo vệ tiến sĩ được 1 năm (dưi sự hưng dẫn của Vakhania). Seminar của chúng tôi (gồm có Vakhania, Chobanyan, V. Tarieladze, Z. Gorgadze, V. Kvaratskhelia, tôi, và sau đã có thêm A. Weron từ Ba Lan sang thực tập) có tên rất hấp dẫn là "Giải tích hàm và lý thuyết xác suất" hoạt động rất tích cực. Có thể nói, tôi đã gặp may khi được làm việc theo nhóm như thế, đặc biệt là làm việc với ChobanyanTarieladze (cả hai sau này đều là những chuyên gia hàng đầu về xác suất trong không gian tuyến tính). Nhưng chúng tôi không ai biết tiếng Pháp, vì thế tôi quyết định học tiếng Pháp để đọc tài liệu, đặc biệt là những bài đăng trong Comptes Rendues của B. Maurey, G. Pisier, Fernique.

Tôi đang nghiên cứu về Hình học các không Banach và thu được một số kết quả theo hướng này, thì đất nước Việt Nam chiến thắng (30/04/1975) và thống nhất. Nửa năm sau (5/10/1975), tôi có mặt tại Hà Nội. Thế rồi, Dacuna Casten (giáo sư Toán học, người Pháp, chuyên ngành về xác suất và thống kê) sang Hà Nội giảng bài (giáo sư Nguyễn Hữu Anh dịch) và đã cập tới không gian lồi đều và trơn đều. Tôi rất hiểu bài giảng của ông và hỏi ông nhiều câu hỏi liên quan đến hình học Banach. Lần đầu tiên tôi được ông cho tôi biết kết quả của Pisier (tuyệt vời) về hình học Banach và ứng dụng vào lý thuyết Martingales Tháng 3/1976, giáo sư Laurent Schwartz đến Hà Nội, thì tôi đã có đã kiến thức nghe và hiễu các bài giảng của ông. Nội dung chính của các bài giảng này là trinh bày các kết quả của seminar Maurey-Schwartz. Ý tưởng của seminar này băt nguồn từ những kết quả của S. Kwapien: ứng dụng lý thuyết toán tử khả tổng tuyệt đại (absolutly summing operators) của Pietsch vào nghiên cứu xác suất. Cụ thể như sau: cho $X,Y$ là hai không gian Banach, và $\mu $ là độ đo xác suất trụ trong $X$ . Giả sử $T:X\rightarrow Y$ là toán tử tuyến tính, liên tục. Khi nào độ đo ảnh $T(\mu)$ là độ đo Radon trên $Y$ ?
Chẳng hạn, khi $X = Y = H$ là không gian Hilbert và $\mu =\gamma$ là độ đo trụ Gauss chuẩn tắc trên $H$, tức là, phiếm hàm đặc trưng có dạng:
$$\widehat{\gamma }=\exp\left (-\frac{\left \| h \right \|^2}{2} \right ),h\in H$$
thì định lý Muorier-Sazonov khẳng định rằng điều kiện cần và đủ để $T(\mu)$ là độ đo Radon là $T$ là tóan tử Hilbert-Schmidt hay tương đương $T$ là toán tử khả tổng tuyệt đại (tức là T biến một dãy khả tổng tuyệt đại yếu thành một dãy khả tổng tuyệt đại mạnh).

Tôi còn nhớ, dù trời nóng, nhưng Ông (lúc đã đã 61 tuổi) giảng bài rất say sưa, rất rõ ràng, khúc triết tại giảng đường C1 của đại học Bách Khoa Hà Nội có tới 100 người nghe. Sau này các bài giảng của Ông được giáo sư Nguyễn Đình Trí và giáo sư Phan Đức Chính ghi chép lại và được in thành sách (bằng tiếng Pháp): Radonifying maps. Thầy Nguyễn Bác Văn và tôi được trực tiếp làm việc với Ông nhiều buổi (lúc thì ở hội trường Bách Khoa, lúc thì ở số 9 Hai Bà Trưng, và cả nơi vợ chồng Ông Bà nghỉ tại khách sạn Metropol Hà Nội. Tôi không nói được tiếng Anh và tiếng Pháp, thành thử thầy Văn đã phiên dịch (rất chính xác) cho tôi. Trong khi tôi trình bày kết quả mới của tôi về Phiễm hàm tuyến tính đo được trong không gian Banach với độ đo Gauss, Ông hỏi tôi đến từng chi tiết nhỏ nhất. Mắt Ông sáng ngời và rất thích kết quả này và Ông nói như khuyên nhủ tôi: "Anh nên học tiếng Pháp rồi sang Pháp làm việc trong seminar của tôi. Anh sẽ được học bổng với số tiền khiêm tốn là 2.500 quan". Tôi còn đặt ra 16 vấn đã về độ đo trong không gian vector topo. Ông chăm chú nghe và cùng tôi thử giải quyết một số vấn đề. Ông làm việc say sưa quên cả giờ giấc, quên cả ăn tối. Lúc bấy giờ tôi hiểu ra rằng "toán học đầy ắp trong Ông", và khi có vấn đã thì từ bộ não của người được giải Fields (lúc 35 tuổi) các ý tưởng toán học tuôn chảy ra mãnh liệt và liên tục. Thỉnh thoảng ông lại hỏi "Anh có hiểu không?"

2) Tôi đã tìm được người hợp tác khoa học, đó là Đặng Hùng Thắng. Có thể nói rằng các bài giảng của giáo sư Laurent Schwartz là cầu nối tôi với sinh viên xuất sắc Đặng Hùng Thắng. Một hôm anh Thắng gặp tôi và trình bày với tôi một khái niệm mới: giá của độ đo xác suất trụ và đặt câu hỏi: giá trụ và giá của độ đo có khi nào trùng nhau không? Tôi thấy câu hỏi rất thú vị. Sau vài ngày suy nghĩ tôi đã tìm được câu trả lời, trong trường hợp giá là tập lồi thì giá trụ và giá của độ đo trùng nhau vì giá trụ là giá của độ đo trong topo yếu. Đặc biệt, điều này Đúng đối với đo đo Gauss, vì giá của độ đo Gauss đối xứng là không gian con. Sau đã tôi đã cho anh Thắng đọc chứng minh của tôi về kết quả này, anh nhận xét rằng chứng minh ấy mở rộng được cho trường hợp độ đo $p$-ổn định với $1<p\leq 2$. Từ đấy tôi và anh Thắng làm việc với nhau và thu được một số kết quả hay về độ đo ổn định.

Lần thứ ba vào đầu năm 1979, Ông giảng về Lý thuyết Martingales và tích phân ngẫu nhiên (giáo sư Nguyễn Đình Ngọc dịch). Lần này Ông trình bày các kết quả chính của Meyer. Lần đầu tiên tôi được biết khái niệm CADLAG từ các bài giảng của Ông và hiểu được tầm quan trọng của khái niệm này. Nhưng ấn tượng hơn cả là tài phiên dịch của giáo sư Nguyễn Đình Ngọc (có lẽ, theo tôi, giáo sư Ngọc là người giỏi nhất về tiếng Pháp và tiếng Việt). Cứ mỗi lần giáo sư Laurent Schwartz vừa giảng hết một ý, thì giáo sư Ngọc đã dịch ngay sang tiếng Việt rất chuẩn mực, tới mức, giáo sư Laurent Schwartz phải thốt lên "Tôi thực sự kinh ngạc về khả năng tiếng Pháp của anh. Tôi chưa bao giờ gặp một người vừa lau bảng vừa dịch rất trôi chảy các điều khóvề toán như ông Ngọc."

Tóm lại, giáo sư Laurent Schwartz có 3 hưóng nghiên cứu chính thì Ông đã giảng cả 3 cho người Việt Nam. Đó là:

1) Lý thuyết về phân phối (hàm suy rộng). Chính nhờ kết quả cơ bản này, Ông đã được giải thưởng Fields năm 1950. Cần chú, ý rằng Sobolev, nhà toán học vĩ đại người Nga, cũng là tác giả của lý thuyết hàm suy rộng, là tác giả của Đnh lý nhúng nổi tiếng trong các không gian Sobolev, được dùng thưòng xuyên trong lý thuyết phương trình đạo hàm riêng.

2) ánh xạ Radon và Hình học các không gian Banach. Thực ra, Ông không phải là người khởi nguồn lý thuyết này, nhưng Ông có ảnh hưởng lớn tới sự phát triển nhanh chóng của lý thuyết này cho tới tận ngày nay nhờ seminar do Ông và Maurey dẫn dắt từ 1969-1981. Đặc biệt, các khái niệm không gian loại $p$ , đối loại $q$; không gian $p$-trơn đều, $q$-lồi đều do Maurey đưa ra (1973) đã được chính MaureyPisier áp dụng rất thành công vào việc nghiên cứu 3 luật cơ bản của lý thuyết xác suất: Luật số Lớn, Định lý giới hạn trung tâm và luật loga lặp.

Cần lưu ý đã có tới 2 giải thưởng Fields trao cho các nhà toán học đạt được những kết quả xuất sắc trong lĩnh vực Hình học các không gian Banach. Đó là Jain Bourgain (1994, Bỉ) và Timothy Gowers (1998, Anh).

3) Tính toán ngẫu nhiên trên đa tạp.

Cuối cùng, một điều rất quan trọng là sở thích của giáo sư Laurent Schwartz: sưu tập tượng và bắt bướm. Bộ sưu tập về loài bướm nhiệt đới của Ông có tới hơn 25.000 loại (là một trong những bộ sưu tập cá nhân lớn nhất), và một số loại mang tên Ông. Khi đến thăm Việt Nam Ông cũng bắt được một số bướm quí hiếm.

Thầy tôi, giáo sư Vakhania (là đệ tử của Sobolev) hết sức kính trọng Sobolev, có lần nói với chúng tôi: "Lịch sử thường bị lãng quên, nhưng lịch sử không bao giờ quên những đóng góp của N. Sobolev". Tôi muốn mượn câu nói này đã viết: "Cho dù lịch sử có phức tạp như thế nào, thì chúng tôi luôn ghi nhớ công lao to lớn của Ông, giáo sư Laurent Schwatsz, đối với sự phát triển toán học Việt Nam".
Gíao sư Nguyễn Đình Trí (đại học Bách Khoa Hà Nội) là người có quan hệ mật thiết với giáo sư Laurent Schwartz từ lần đầu tiên Ông sang thăm Việt Nam và hai lần sau đã Ông đều giảng bài ở đại học Bách Khoa Hà Nội. Trong một thư (message) giáo sư Trí gửi cho tôi (ngày 20/06/2012) có đoạn cảm động sau:

"Toi doc bai ong viet ve GS L. Schwartz va rat hoan nghenh bai viet nay. Toi la nguoi tiep xuc voi GS Scwartz ngay tu lan Gs sang VN lan dau, sau do gap GS o ICM Moscow 1966 va cung co nhung hoat dong rat hay tai do voi nhung nha toan hoc co cam tinh coi VN, do GS Schwartz goi y va chu tri. Ong con sang lan cuoi cung do loi moi cua Bo truong Tran Hong Quan de tham van ve viec danh gia cac truong dai hoc VN, vi luc do o Phap moi thanh lap Comite National d'Evaluation ma GS Schwartz lam chu tich dau tien. Toi cung da viet ve nhung van de do ngay sau hki GS Schwartz mat. Toi thay anh viet nhu the la du. Toi cung con duoc ve tham mo GS Schwartz o lang Eden ma anh Khoai da viet. O day co mot mo chung cua GS cung voi con trai ong".

Gíao sư Đặng Hùng Thắng gửi 2 thư (message) cho tôi viết:
Trong thư thứ nhất (ngày 17/06/2012)
1) Lần thứ nhất đến VN mùa hè năm 1968: Lúc đó em đang là học sinh lớp 8 chuyên Toán ĐHSP Hà Nội (tức lớp 10 bây giờ). Em đang ở chỗ sơ tán thì được Nhà trường cử về HN tham dự buổi nói chuyện của của L.Sha tại Giảng đường lớn ĐHTH (nay là hội trường Lê Văn Thiêm) với các học sinh giỏi Toán tiêu biểu của Thủ đô. Hôm đó có sự hiện diện của Thủ tướng Phạm Văn Đồng và Bộ trường Tạ Quang Bửu. Em còn nhớ GS đến bắt tay em và một số bạn học sinh và nói “Làm toán là một công việc rất thú vị

2) Lần thứ hai: Thời gian là từ 8/3/1976 đến 26/3 1976 tại Hội trường trường ĐHBK Hà Nội (không phải là tháng 8/1976 như trong bài) “Người VN không quên tôi, Việt nam ghi dấu ấn trong cuộc đời tôi”. Em (và anh) chính là một trong những người VN mãi mãi không quên ông. Bài giảng tuyệt diệu của GS đã thực sự ảnh hưng rất lớn với em, khi đã vừa tốt nghiệp ĐH, đã ghi dấu ấn,tạo bước ngoặt trong cuộc đì làm toán của em: Những bài giảng của ông về Xác suất Radon, xác suất trụ, thác triển độ đo trụ thành độ đo Radon, ,tính chất hình học (loại và đối loại) của không gian Banach, mối quan hệ giũa tính chất hình học của không gian Banach và tính chất xác suất, ánh xạ $p$-tổng hóa và ánh xạ Radon hóa, áp dụng vảo chuyển động Brown… đã cuốn hút em, làm em thực sự thú vị và làm nền tảng cho kiến thức của em về Xác suất, Độ đo, tích phân, Giải tích hàm mãi cho đến bây giờ.

3) Lần thứ ba : Cuối năm 1979 (không phaỉ là tháng 8/1976 như trong bài) Em và anh đã cho ông đọc bài báo trình bày những kết quả mới của mình về thác triển độ đo trụ $p$- ổn định thành xác suất Radon. Từ Pháp GS đã đc gửi thư trả lời (qua địa chỉ Bộ trưởng Nguyễn Đình Tứ ) nêu ra những góp ý comment của ông. Không những vậy ông còn nhờ G.Pisier đc và gửi kèm theo các góp ý và comment của G.Pisier về các kết quả này.

Trong thư thứ hai (ngày 23/06/2012)

Anh Tiến thân kính
Em tìm lại các vở ghi chép cũ thì xác định được chính xác thời điểm đến VN giảng bài lần thứ ba: Từ ngày 11/5/1979 đn ngày 15/5/1979 Nội dung về Lý thuyết Martingale , tích phân ngẫu nhiên và ứng dụng trong giải tích, PT đạo hàm riêng bài toán biên... Lần này GS trình bày tổng quan, không phải bài giảng chi tiết như lần thứ hai (từ 8/3/1976 đn 26/3/1976).


Em cũng muốn bổ sung thêm về các giai đoạn nghiên cứu của GS: Giai đoạn thứ nhất (1944-1954): Lý thuyết phân bố, hàm suy rộng. Giai đoạn thứ hai (từ 1954-1966): Giải tích và Phương trình đạo hàm riêng (PDE). Giai đoạn thứ ba ( từ 1967-1988): Lý thuyết xác suất và hình học không gian Banach.

Ngoài ra GS còn nhiều công trình về lịch sử Toán học và giảng dạy Toán học. Cuốn hồi ký khá dày của GS, em đã được đc bản dich ra tiếng Anh ở một thư viện nước ngoài, có nhiều đoạn cảm động GS viết về VN, thể hiện tình cảm yêu mến của ông đối với đất nước và con người VN. Ước gì cuốn hồi ký này được dịch toàn bộ hoặc một phần ra tiếng Việt thì đây là một việc làm rất có ý nghĩa tưởng nhớ tới GS



Hà Nội, hè 2012.

Hình đã gửi


Như vậy, giáo sư Laurent Schwartz có 16 "học trò" và 2278 "môn đệ".

(GS Nguyễn Duy Tiến)


#330730 Toán học trong bầu cử

Gửi bởi Ban Biên Tập trong 01-07-2012 - 09:42

Trong một chuyện cổ tích, có một cô gái có 3 chàng trai tài ba đến cầu hôn, mà cô gái loay hoay mãi vẫn không biết nên chọn tràng nào. Cô thích chàng B hơn chàng A, vì chàng B hào hoa phong nhã hơn. Nhưng cô lại thích chàng C hơn chàng B, vì chàng C thông minh sắc sảo hơn, nhưng cô lại thích chàng A hơn chàng C, vì chàng A vạm vỡ chắc chắn hơn. Mãi không quyết định được, nên cô đành ở vậy.

Cô gái trong chuyện trên gặp phải tình huống éo le, mà theo thuật ngữ toán học thì gọi là các lựa chọn không xếp được theo thứ tự có tính bắc cầu (tức là nếu thích A hơn B, thích B hơn C, thì cũng thích A hơn C), mà việc xếp thứ tự bị mắc phải lỗi quay vòng (cyclicity). Theo thuật ngữ xã hội, thì lỗi quay vòng của các lựa chọn của cô gái này được gọi là phi lý trí.

Từng người một có thể phi lý trí, thì xã hội cũng có thể phi lý trí. Tệ hơn nữa, là kể cả khi từng người một trong xã hội đều có lý trí (tức là biết mình thích lựa chọn nào hơn lựa chọn nào trong số các lựa chọn được đưa ra), thì cả xã hội cũng vẫn có thể phi lý trí. Nghịch lý về lựa chọn xã hội này đã được Condorcet phát hiện từ thế kỷ 18, và hay được gọi là nghịch lý Condorcet.

Lấy một ví dụ đơn giản sau: giả sử có 5 người $A,B, C,D,E$ cần chọn ra một giải pháp trong số 3 giải pháp $x,y,z$. Kết quả chọn lựa như sau:

$A: x,y,z$ $($tức là $A$ thích $x$ nhất, sau đó đến $y$, và cuối cùng mới đến $z$$)$
$B: y,z,x$
$C: z,x,y$
$D: y,z,x$
$E: x,z,y$

Có thể coi là xã hội 5 thành viên $A, B, C, D, E$ thích lựa chọn $x$ hơn là $y$, vì có 3 người thích $x$ hơn $y$ trong khi chỉ có 2 người thích $y$ hơn $x$. Nhưng nếu so giữa $y$ và $z$ thì $y$ thắng vì có 3 người thích $y$ hơn $z$, và nếu so giữa $z$ và $x$ thì $z$ lại thắng. Như vậy là cái xã hội này đã rơi vào vòng phi ý trí, như là cô gái phía trên.

Đến quãng những năm 1950-1960, vấn đề phi lý trí này đã được Kenneth Arrow nghiên cứu và đưa ra thành định lý về tính "không thể có lý trí" của các hệ thống bầu cử (chọn lựa) trong xã hội. Một phần nhờ định lý này mà Arrow được giải Nobel về kinh tế năm 1972.

Phát biểu toán học của định lý không thể (impossibility theorem) của Arrow như sau:
Gọi A là tập hợp các lựa chọn (các ứng cử viên), và $L(A)$ là tập hợp tất cả các xắp xếp thứ tự tuyến tính đầy đủ của $A$. Một xắp xếp thứ tự tuyến tính đầy đủ là khi với 2 phần tử $x,y$ khác nhau bất kỳ của $A$ thì hoặc $x > y$ ($x$ được thích hơn $y$) hoặc $y > x$ nhưng không thể cả hai, và có tính bắc cầu. Một hàm lựa chọn xã hội chặt chẽ là một hàm $F:L{\left( A \right)^N} \to L\left( A \right)$, trong đó $N$ là số cử tri (mỗi cử tri xắp xếp các lựa chọn theo ý mình, và hàm $F$ là hàm bầu cử đưa ra lựa chọn chung của toàn xã hội dựa trên các lựa chọn riêng của từng cử tri). Định lý Arrow nói rằng, nếu như tập $A$ có ít nhất 3 phần tử (3 ứng cử viên), thì không thể có được hàm $F$ nào thỏa mãn cả 3 điều kiện sau:

1) Tính nhất quán, hay còn gọi là điều kiện hiệu quả Pareto) (unanimity, Pareto efficiency): Nếu mọi cử tri đều cho $x$ nằm trên $y$, thì trong lựa chon chung của xã hội $x$ cũng nằm trên $y$.

2) Không có độc tài (no dictator): Có độc tài là khi tồn tại một chỉ số $i \in \left\{ {1,2,...,N} \right\}$ sao cho luôn có $F\left( {{R_1},{R_2},...,{R_n}} \right) = {R_i}$ với mọi $\left( {{R_1},{R_2},...,{R_n}} \right) \in L{\left( A \right)^N}$ (độc tài quyết định thay cho toàn xã hội, bất kể sở thích của những người khác ra sao)

3) Sự không phụ thuộc vào các lựa chọn không liên quan (independence of irrelevant alternatives, viết tắt là IIA). Điều đó có nghĩa là, nếu như các cử tri chuyển vị trí xếp hạng của một lựa chọn $z$ nào đó trong các bảng xếp hạng của mình nhưng không làm thay đổi thứ tự tương đối của các lựa chọn khác, thì điều đó cũng không làm thay đổi thứ tự tượng đối của các lựa chọn khác trong bảng xếp hạng chung của toàn xã hội. ($z$ ở đây được coi là lựa chọn không liên quan đến bảng xếp hạng tương đối giữa các lựa chọn khác). Tính chất này có thể hình dung qua ví dụ sau: Một người được chọn giữa 1 xe BMW và 1 xe Mercedes và quyết định chọn BMW, sau khi tham khảo nhiều ý kiến. Khi người đó nghe nói "Audi bền lắm", thì điều đó không làm thay đổi quyết định chọn BMW thay vì Mercedes.

Hàm $F$ phía trên là luật bầu cử (lựa chọn xã hội). Như vậy, theo định lý Arrow, thì không có một hệ thống bầu cử nào thỏa mãn cả 3 tính chất rất có lý trên, mà lại luôn xắp xếp được thứ tự các lựa chọn xã hội, mà không rơi vào tình trạng quay vòng phi lý trí dù cho các cử tri chọn lựa ra sao. Chứng minh định lý Arrow không khó: nó chỉ dài khoảng 1 trang, và có thể làm bài tập cho những ai tò mò. Còn ai sốt ruột thì có thể đọc chứng minh của nó ở trang web wikipedia về định lý Arrow đã có link phía trên. Tuy không khó, nhưng định lý Arrow được coi là một trong các cột mốc quan trọng nhất của lý thuyết hiện đại về lựa chọn của xã hội. Nó cũng hay bị suy diễn và hiểu sai. Chẳng hạn, người ta suy diễn từ nó ra rằng các hệ thống dân chủ đều tồi, không có một hệ thống bầu cử dân chủ nào là tốt cả. Thực ra, định lý Arrow không phải vậy. Nó chỉ nói rằng, với bất cứ một hệ thống bầu cử nào theo kiểu sắp xếp thứ tự dựa trên sắp xếp thứ tự của các cử tri, thì cũng tồn tại những tình huống "éo le" không cho ra kết quả thỏa mãn các tính chất "có lý trí". Nhưng các tình huống “éo le” này ít xảy ra trên thực tế, và nếu nó éo le với hệ thống bầu cử này, thì có thể là không "éo le" với hệ thống khác, nên việc chọn lựa hệ thống bầu cử thích hợp cũng là quan trọng.

Một "định lý không thể" thú vị khác trong kinh tế học là định lý Holstrom, được phát biểu như sau: không tồn tại "incentive system" nào thỏa mãn cả 3 tính chất:

1) Cân bằng ngân sách (income = outflow)

2) Có điểm ổn định Nash

3) Thỏa mãn điều kiện hiệu quả Pareto.


#329852 Tổng kết Kì thi thử Đại học năm 2012 của Diễn đàn Toán học (VMF)

Gửi bởi Ban Biên Tập trong 27-06-2012 - 23:56

logo8namvmf.jpg



1. Giới thiệu

Kì thi thử Đại học VMF - 2012 diễn ra từ thứ 7 ngày 12/11/2011 và kết thúc vào thứ 5 ngày 31/05/2012. Đây là lần đầu tiên Diễn Đàn Toán Học (VMF) tổ chức một cuộc thi hướng vào đối tượng là các học sinh phổ thông lớp 12 chuẩn bị thi Đại học, với mong muốn chuẩn bị cho các sĩ tử một hành trang kiến thức vững chắc trước khi bước vào "trận đánh thật" sẽ diễn ra vào tháng 7 tới. Xuyên suốt hơn 5 tháng, diễn đàn đã soạn thảo bộ 7 đề thi môn toán theo chương trình của Bộ Giáo Dục nhằm giúp các bạn thí sinh ôn tập. Với mỗi đề thi, các thành viên Diễn Đàn Toán Học sẽ có 1 tuần kể từ khi đề thi được công bố để giải quyết hết khả năng của mình trước khi nộp bài làm cho Ban Giám Khảo (BGK). Người tham gia thi thử chỉ được nộp bài một lần duy nhất. Hết hạn nộp bài đề thi sẽ được thảo luận rộng rãi trên diễn đàn, tất cả các thành viên sẽ được xem và nhận xét bài làm của các thí sinh cũng như chất lượng đề thi. BGK gồm 6 người, là các thầy giáo toán, sinh viên, nghiên cứu sinh đang hoạt động tích cực trên diễn đàn : WWW, hxthanh, E. Galois, batigoal, ongtroi, T*genie*. BGK có trách nhiệm ra đề thi, chấm bài và giải đáp thắc mắc để đảm bảo kì thi được diễn ra tốt đẹp. Ngoài đề thi số 1, 6 đề thi còn lại đều ra đúng cấu trúc như một đề thi thật. Tuy còn nhiều thiếu sót, số lượng tham gia mỗi đề thi chưa đông nhưng kì thi thử Đại học phần nào cũng tạo được tiếng vang và là một hoạt động tương đối thành công của ban điều hành VMF kể từ khi diễn đàn quay trở lại với một giao diện mới. Hi vọng rằng kì thi thử sẽ là một bước đệm thu hút các bạn yêu toán, các thầy giáo toán đến với ngôi nhà chung VMF.

2. Một vài số liệu

7 đề thi thử của VMF đã nhận được sự quan tâm lớn của độc giả với tổng cộng 6633 lượt tải đề tính đến hết ngày 21/06/2012 (trung bình gần 1000 lượt tải mỗi đề thi), trong đó đề số 1 có số lượng tải nhiều nhất (1449 lượt) và đề số 7 ít nhất (464 lượt). Đề số 5 có số lượng thí sinh tham gia đông nhất (8 thí sinh) và thấp nhất là đề số 7 chỉ có 1 thí sinh tham gia gửi bài. Thí sinh tham gia tích cực nhất là bạn Đoàn Quốc Khánh (khanh3570883). Khánh tham gia 6/7 đề thi, đặc biệt em mới chỉ là học sinh lớp 11.

$$\begin{array}{|c|c|c|}
\hline
\textbf{Đề thi thử số} & \textbf{Lượt tải về} & \textbf{Số thí sinh tham gia} \\
\hline
1 & 1449 & 3\\
\hline
2 & 1145 & 7 \\
\hline
3 & 1149 & 5 \\
\hline
4 & 700 & 7 \\
\hline
5 & 852 & 8 \\
\hline
6 & 874 & 3 \\
\hline
7 & 464 & 1 \\
\hline
\end{array}$$


Bảng tổng kết thi thử VMF - 2012

vmf1.jpg

Đồ thị cột biểu diễn số lượt tải về qua mỗi đề thi


vmf2.jpg

Đồ thị cột biểu diễn số thí sinh tham gia qua mỗi đề thi



3. Một vài link trao đổi

Các bạn có thể trao đổi về các đề thi, tham khảo đáp án của BGK cũng như đáp án của các thí sinh tham gia tại các link sau :


$\bullet$ Đề thi thử số 1 : http://diendantoanho...showtopic=64684

$\bullet$ Đề thi thử số 2 : http://diendantoanho...showtopic=65834

$\bullet$ Đề thi thử số 3 : http://diendantoanho...showtopic=67158

$\bullet$ Đề thi thử số 4 : http://diendantoanho...showtopic=68420

$\bullet$ Đề thi thử số 5 : http://diendantoanho...showtopic=69292

$\bullet$ Đề thi thử số 6 : http://diendantoanho...showtopic=70913

$\bullet$ Đề thi thử số 7 : http://diendantoanho...showtopic=72545


4. Giải thưởng

Ngoài việc trao thưởng cho thí sinh có tổng điểm cao nhất qua 7 đề thi thử, Diễn Đàn Toán Học nhân kỉ niệm tròn 8 năm tuổi cũng trao giải thưởng là 3 chiếc áo đồng phục VMF cho 3 bạn giải quyết xuất sắc nhất đề thi thử số 3.

4.1. Danh sách các bạn được giải ở đề thi thử số 3

$$\begin{array}{|c|c|c|c|}
\hline
\color{Red}{\textbf{Tên}} & \color{Red}{\textbf{Nick trên diễn đàn}} & \color{Red}{\textbf{Điểm đề thi thử số 3/10}} & \color{Red}{\textbf{Hạng}} \\
\hline
\textbf{Phạm Hùng Vương} & \text{h.vuong_pdl} & 8.75 & 1 \\
\hline
\textbf{Thái Nguyễn Hưng} & \text{Nguyễn Hưng} & 8.5 & 2 \\
\hline
\textbf{Đoàn Quốc Khánh}^{\star} & \text{khanh3570883} & 8.5 & 3 \\
\hline
\end{array}$$

($\star$) Khánh có cùng số điểm với Hưng nhưng xếp sau do xét hệ số phụ (nộp bài muộn hơn).

4.2. Danh sách các bạn được giải chung cuộc

$$\begin{array}{|c|c|c|c|c|}
\hline
\color{Red}{\textbf{Tên}} & \color{Red}{\textbf{Nick trên diễn đàn}} & \color{Red}{\textbf{Số đề tham gia giải/7}} & \color{Red}{\textbf{Tổng điểm/70}} & \color{Red}{\textbf{Hạng}} \\
\hline
\textbf{Đoàn Quốc Khánh} & \text{khanh3570883} & 6 & 48.75 & 1 \\
\hline
\textbf{Vũ Đình Việt} & \text{vietfrog} & 5 & 42.25 & 2 \\
\hline
\textbf{Phạm Hùng Vương} & \text{h.vuong_pdl} & 5 & 41.5 & 3 \\
\hline
\end{array}$$

5. Lời cảm ơn

BGK thi thử Đại học VMF - 2012 xin gửi lời cảm ơn đến bạn PSW đã gửi bài cộng tác cho chúng tôi trong đề thi thử số 3 và admin Nesbit đã duy trì những vấn đề kĩ thuật cho BGK trong suốt đợt thi thử.


6. Lời kết


Graduation.jpg


Thành công là hành trình chứ không chỉ là điểm đến...


Như vậy là kì thi thử Đại học môn toán năm 2012 của VMF đã thành công tốt đẹp. Do lần đầu tổ chức vẫn còn tồn đọng nhiều hạn chế như chất lượng đề thi, công tác chấm bài... nhưng chúng ta cũng cần ghi nhận những nỗ lực của tập thể BGK, BQT diễn đàn cũng như tất cả các thí sinh tham gia. Diễn Đàn Toán Học chúc tất cả các thí sinh VMF nói riêng và tất cả các thí sinh lớp 12 chuẩn bị bước vào kì thi đại học nói chung một mùa thi thành công và gặp nhiều may mắn. Hi vọng 7 đề thi thử của chúng tôi sẽ góp một phần nhỏ hành trang kiến thức giúp các bạn vượt ải vũ môn đầy cam go vào tháng 7 tới. Sự cổ vũ động viên của các bạn là nguồn cổ vũ tinh thần to lớn để chúng tôi tiếp thục hoàn thành các bộ đề thi thử sau được tốt hơn, đóng góp nhiều hơn nữa vào kho tài liệu chung cho các bạn yêu toán phổ thông.


Trân trọng.


Thay mặt BGK thi thử Đại học 2012,

T*genie*




#329285 Toán học trong bầu cử

Gửi bởi Ban Biên Tập trong 26-06-2012 - 10:11

Nicolas de Condorcet (1743-1794) là một nhà triết học, toán học và chính trị học có ảnh hưởng rất lớn ở Pháp thời phục hưng. Có một khái niệm trong chính trị học (lý thuyết về lựa chọn trong xã hội), gọi là "Condorcet winner", mang tên ông ta.

Giả sử có một cuộc bỏ phiếu (bầu cử) để chọn ra một trong $(>= 2)$ các lựa chọn (ứng cử viên) khác nhau: $A,B,C, … , Z$. Lựa chọn $Z$ sẽ được gọi là "Condorcet winner" (kẻ thắng theo nghĩa Condorcet) nếu như khi "đấu tay đôi" với bất kỳ lựa chọn khác nào thì $Z$ đều thắng: giữa $Z$ và $A$ (hay $B, C$, …) thì có hơn $\dfrac{1}{2}$ số người bỏ phiếu thích bầu cho $Z$ hơn là cho $A$ (hay $B, C$, …). "Condorcet loser" được định nghĩa một cách tương tự.

Giả sử có 1 luật bầu cử, áp dụng cho 1 cuộc bầu cử để chọn ra đúng 1 người (hay 1 lựa chọn) thắng. Luật đó sẽ được gọi là thỏa mãn tính Condorcet, nếu như trong mọi trường hợp mà có 1 Condorcet winner thì Condorcet winner đó thắng cử, và ngược lại mọi Condorcet loser đều không thắng cử. (Dễ thấy là không thể có quá 1 Condorcet winner, và có nhiều tình huống không có Condorcet winner nào cả).

Nhiều hệ thống bầu cử được dùng trên thế giới hiện tại không thỏa mãn tính Cordorcet. Trong đó "tai tiếng nhất" là hệ thống "pure plurality": chỉ bầu 1 vòng, và ai có nhiều số phiếu nhất trong số các ứng cử viên (không nhất thiết phải đạt trên $50\%$ số phiếu) thì thắng cuộc. Một ví dụ là cuộc thi trao giải âm nhạc Grammy Award (giải danh giá nhất ở Mỹ) năm 1985: album tương đối mờ nhạt "Can’t Slow Down" của Lionel Richie được giải năm đó, trong khi hai album ứng cử viên xuất chúng năm đó là "Born in the USA" của Bruce Springsteen và "Purple Rain" của Prince được mọi người ưa chuộng hơn thì lại "xâu xé nhau" để cuối cùng cả hai đều được ít phiếu hơn Richie. Theo bình luận viên, thì nếu bầu theo đôi một, chắc chắn Richie sẽ thua cả hai album kia. Ngay hệ thống bầu cử tổng thống 2 vòng của Pháp cũng không tránh khỏi "bi kịch" này: vào năm 2002, ông Jospin, người được đánh giá là được ưa chuộng nhất trong số các ứng cử viên lúc đó, đã bị loại ngay vòng đầu vì phiếu bị phân tán sang những ứng cử viên "gần phe" với ông ta, để cho ứng cử viên đảng cực hữu lọt vào vòng hai.

Tuy nhiên, có một câu hỏi "lý thuyết" là: liệu việc thỏa mãn tính Condorcet có chắc là tốt đối với một hệ thống bầu cử không? Và câu trả lời là KHÔNG! Nói cách khác, lựa chọn là Condorcet winner cũng có những khi lại là lựa chọn rất tồi đối với xã hội, và ngược lại Condorcet loser có khi lại là lựa chọn tốt cho xã hội.

Thử lấy một ví dụ sau: $3$ người "cannibal" $A, B, C$ có $4$ lựa chọn $a, b, c, d$. Lựa chọn $a$ là $B$ và $C$ ăn thịt $A$, lựa chọn $b$ là $A$ và $C$ ăn thịt $b$, lựa chọn $c$ là $A$ và $B$ ăn thịt $C$, và lựa chọn $d$ là không ăn thịt nhau mà tiến hóa lên tìm cái khác để ăn. Những người cannibal này thích ăn thịt người khác và không thích mình bị ăn thịt. Như vậy lựa chọn $d$ là Condorcet loser ở đây, nhưng có khi lại là lựa chọn tốt nhất cho nhóm người $A, B, C$. Nếu chẳng hạn $A, B, C$ tính điểm cho mỗi lựa chọn theo sự sung sướng hay đau khổ mà nó mang lại cho mình: ví dụ như $A$ cho điểm $-10$ cho lựa chọn $a$ (quá đau khổ khi bị ăn thịt), và $+1$ cho mỗi lựa chọn $b$ và $c$, và $0$ cho lựa chọn $d$, và $B$ và $C$ cũng làm tương tự, rồi cộng các điểm vào với nhau, thì lựa chọn $d$ sẽ là lựa chọn có tổng số điểm cao nhất $(= 0)$ trong khi các lựa chọn khác có tổng số điểm là âm.

(Ví dụ Grammy Award phía trên là lấy từ bài viết của Paul E. Johnson: Voting systems, 2005)

Theo zung.zetamu.net




#326543 Tìm số hạng tổng quát của dãy số bằng hàm Hypebolic

Gửi bởi Ban Biên Tập trong 18-06-2012 - 08:08

Ví dụ 4: Tìm số hạng tổng quát của dãy số $({u_n})$ định bởi: $\left\{ \begin{array}{l} {u_1} = \frac{5}{4}\\ {u_{n + 1}} = 2{u_n} + \sqrt {3u_n^2 - 3} \,,\,\,\forall n \in \mathbb{N^*} \end{array} \right.$


Giải: Đặt $\left\{ \begin{array}{l} ch\beta = 2\\ sh\beta = \sqrt 3 \end{array} \right.$. Giải hệ ta được ${e^\beta } = 2 + \sqrt 3 $

Đặt $\left\{ \begin{array}{l} ch\alpha = \frac{5}{4}\\ sh\alpha = \sqrt {{{\left( {\frac{5}{4}} \right)}^2} - 1} = \frac{3}{4} \end{array} \right.$. Giải hệ ta được ${e^\alpha } = 2$

Ta có: ${u_1} = ch \alpha$. Giả sử ${u_n} = ch\left[ {\alpha + \left( {n - 1} \right)\beta } \right]$. Khi đó: $$ch\beta .ch\left[ {\alpha + \left( {n - 1} \right)\beta } \right] + sh\alpha .sh\left[ {\alpha + \left( {n - 1} \right)\beta } \right] = ch\left( {\alpha + n\beta } \right)$$
Vậy: $${u_n} = \frac{{{e^{\alpha + \left( {n - 1} \right)\beta }} + {e^{ - \,\alpha - \,\left( {n - 1} \right)\beta }}}}{2} = {\left( {2 + \sqrt 3 } \right)^{n - 1}} + \frac{1}{4}{\left( {2 - \sqrt 3 } \right)^{n - 1}}$$

Hình đã gửi


Ví dụ 5: Tìm số hạng tổng quát của dãy số $({u_n})$ định bởi: $\left\{ \begin{array}{l} {u_1} = 3\\ {u_{n + 1}} = \frac{1}{2}{u_n}\sqrt {16 + u_n^2} \,,\,\,\forall n \in \mathbb{N^*} \end{array} \right.$

Giải: Biến đổi giả thiết: $\frac{{{u_{n + 1}}}}{4} = 2\left( {\frac{{{u_n}}}{4}} \right)\sqrt {1 + {{\left( {\frac{{{u_n}}}{4}} \right)}^2}} $

Đặt ${x_n} = \frac{{{u_n}}}{4}$ ta được dãy $({x_n})$ định bởi: $\left\{ \begin{array}{l} {x_1} = \frac{3}{4}\\ {x_{n + 1}} = 2{x_n}\sqrt {1 + x_n^2} \,,\,\,\forall n \in \mathbb{N^*} \end{array} \right.$

Đặt $sh\alpha = \frac{3}{4} \Leftrightarrow \frac{{{e^\alpha } - {e^{ - \,\alpha }}}}{2} = \frac{3}{4}$. Giải phương trình ta được ${e^\alpha } = 2$

Ta có: ${x_1} = sh\alpha $. Giả sử ${x_n} = sh\left( {{2^{n - 1}}\alpha } \right)$. Khi đó: $${x_{n + 1}} = 2sh\left( {{2^{n - 1}}\alpha } \right)\sqrt {1 + s{h^2}\left( {{2^{n - 1}}\alpha } \right)} = 2sh\left( {{2^{n - 1}}\alpha } \right)ch\left( {{2^{n - 1}}\alpha } \right) = sh\left( {{2^n}\alpha } \right)$$
Vậy: $${x_n} = sh\left( {{2^{n - 1}}\alpha } \right) = \frac{{{{\left( {{e^\alpha }} \right)}^{{2^{n - 1}}}} - {{\left( {{e^{ - \,\alpha }}} \right)}^{{2^{n - 1}}}}}}{2}\,\,\,\text{ suy ra}\,\,\, {u_n} = 2\left( {{2^{{2^{n - 1}}}} - {{\left( {\frac{1}{2}} \right)}^{{2^{n - 1}}}}} \right)$$
Ví dụ 6: Tìm số hạng tổng quát của dãy số $({u_n})$ định bởi: $\left\{ \begin{array}{l} {u_1} = \frac{3}{{\sqrt 6 }}\\ {u_{n + 1}} = 24u_n^3 - 12\sqrt 6 u_n^2 + 15{u_n} - \sqrt 6 \,,\,\,\forall n \in \mathbb{N^*} \end{array} \right.$

Nhận xét: Đặt ${u_n} = a{x_n} + b$, thay vào giả thiết và rút gọn ta được: $$a{x_{n + 1}} + b = 24{a^3}x_n^3 + 12\left( {6{a^2}b - \sqrt 6 {a^2}} \right)x_n^2 + 3\left( {24a{b^2} - 8\sqrt 6 ab + 5a} \right){x_n} + 24{b^3} - 12\sqrt 6 {b^2} + 15b - \sqrt 6 $$
Đầu tiên ta chọn $b$ sao cho: $\left\{ \begin{array}{l} 6{a^2}b - \sqrt 6 {a^2} = 0\\ 24{b^3} - 12\sqrt 6 {b^2} + 15b - \sqrt 6 = b \end{array} \right. \Rightarrow b = \frac{1}{{\sqrt 6 }}$

Khi đó: $a{x_{n + 1}} = 24{a^3}x_n^3 + 3a{x_n}\,\,\text{hay}\,\,{x_{n + 1}} = 24{a^2}x_n^3 + 3{x_n}$

Giải: Đặt ${u_n} = \frac{1}{{\sqrt 6 }}{x_n} + \frac{1}{{\sqrt 6 }}$ ta được dãy số $({x_n})$ định bởi: $\left\{ \begin{array}{l} {x_1} = 2\\ {x_{n + 1}} = 4x_n^3 + 3{x_n} \end{array} \right.$

Đặt $sh\alpha = 2 \Leftrightarrow \frac{{{e^\alpha } - {e^{ - \,\alpha }}}}{2} = 2$. Giải phương trình ta chọn nghiệm ${e^\alpha } = 2 + \sqrt 5 $

Áp dụng công thức $sh\left( {3\alpha } \right) = 4s{h^3}\alpha + 3sh\alpha $, rồi chứng minh quy nạp ta được:
$${x_n} = sh\left( {{3^{n - 1}}\alpha } \right) = \frac{{{{\left( {{e^\alpha }} \right)}^{{3^{n - 1}}}} - {{\left( {{e^{ - \,\alpha }}} \right)}^{{3^{n - 1}}}}}}{2} = \frac{{{{\left( {2 + \sqrt 5 } \right)}^{{3^{n - 1}}}} - {{\left( {\dfrac{1}{{2 + \sqrt 5 }}} \right)}^{{3^{n - 1}}}}}}{2}$$
Vậy: ${u_n} = \frac{1}{{2\sqrt 6 }}\left[ {{{\left( {2 + \sqrt 5 } \right)}^{{3^{n - 1}}}} - {{\left( {\frac{1}{{2 + \sqrt 5 }}} \right)}^{{3^{n - 1}}}}} \right] + \frac{1}{{\sqrt 6 }}$

Ví dụ 7: Tìm số hạng tổng quát của dãy số $({u_n})$ định bởi: $\left\{ \begin{array}{l} {u_1} = 1\\ {u_{n + 1}} = \frac{{ - \,u_n^2}}{{u_n^2 + 2{u_n} + 2}}\,,\,\,\forall n \in \mathbb{N^*} \end{array} \right.$

Giải: Biến đổi giả thiết: ${u_{n + 1}} + 1 = \frac{{2\left( {{u_{n + 1}} + 1} \right)}}{{1 + {{\left( {{u_{n + 1}} + 1} \right)}^2}}}$

Đặt ${x_n} = {u_n} + 1$ ta được dãy số $({x_n})$ định bởi: $\left\{ \begin{array}{l} {x_1} = 2\\ {x_{n + 1}} = \frac{{2{x_n}}}{{1 + x_n^2}}\,,\,\,\forall n \in \mathbb{N^*} \end{array} \right.$

Đặt $\coth \alpha = 2 \Leftrightarrow \frac{{{e^\alpha } + {e^{ - \,\alpha }}}}{{{e^\alpha } - {e^{ - \,\alpha }}}} = 2$. Giải phương trình ta được: ${e^\alpha } = \sqrt 3 $

Ta có: ${x_2} = \frac{{2\coth \alpha }}{{1 + {{\coth }^2}\alpha }} = \frac{{2\frac{1}{{th\alpha }}}}{{1 + \frac{1}{{t{h^2}\alpha }}}} = \frac{{2th\alpha }}{{1 + t{h^2}\alpha }} = th\left( {2\alpha } \right)$

Giả sử ${x_n} = th\left( {{2^{n - 1}}\alpha } \right)$. Khi đó: ${x_{n + 1}} = \frac{{2th\left( {{2^{n - 1}}\alpha } \right)}}{{1 + t{h^2}\left( {{2^{n - 1}}\alpha } \right)}} = th\left( {{2^n}\alpha } \right)$

Vậy: ${x_n} = th\left( {{2^{n - 1}}\alpha } \right) = \frac{{{{\left( {{e^{2\alpha }}} \right)}^{{2^{n - 1}}}} - 1}}{{{{\left( {{e^{2\alpha }}} \right)}^{{2^{n - 1}}}} + 1}} = \frac{{{3^{{2^{n - 1}}}} - 1}}{{{3^{{2^{n - 1}}}} + 1}}\,\,\,\text {suy ra}\,\,\,\,{u_n} = \frac{{ - \,2}}{{{3^{{2^{n - 1}}}} + 1}},\,\,\,\forall n \ge 2$

Ví dụ 8: Tìm số hạng tổng quát của dãy số $({u_n})$ định bởi: $\left\{ \begin{array}{l} {u_1} = \frac{5}{4}\\ {u_{n + 1}} = \frac{{u_n^3 + 3{u_n}}}{{1 + 3u_n^2}}\,,\,\,\forall n \in \mathbb{N^*} \end{array} \right.$

Giải: Đặt $\coth \alpha = {u_1} \Leftrightarrow \frac{{{e^\alpha } + {e^{ - \,\alpha }}}}{{{e^\alpha } - {e^{ - \,\alpha }}}} = \frac{5}{4}$. Giải phương trình ta được: ${e^\alpha } = 3$

Giả sử ${u_n} = \coth \left( {{3^{n - 1}}\alpha } \right)$. Khi đó: ${u_{n + 1}} = \frac{{{{\coth }^3}\left( {{3^{n - 1}}\alpha } \right) + 3\coth \left( {{3^{n - 1}}\alpha } \right)}}{{1 + 3{{\coth }^2}\left( {{3^{n - 1}}\alpha } \right)}} = \dfrac{{\dfrac{1}{{t{h^3}\left( {{3^{n - 1}}\alpha } \right)}} + \dfrac{3}{{th\left( {{3^{n - 1}}\alpha } \right)}}}}{{1 + \dfrac{3}{{t{h^2}\left( {{3^{n - 1}}\alpha } \right)}}}}$

Vậy: ${u_n} = \coth \left( {{3^{n - 1}}\alpha } \right) = \frac{{{9^{{3^{n - 1}}}} + 1}}{{{9^{{3^{n - 1}}}} - 1}}$

Ví dụ 9: Cho dãy $({x_n})$ định bởi: $\left\{ \begin{array}{l} {x_1} = 4\\ {x_{n + 1}} = x_n^2 - 2\,,\,\,\forall n \in \mathbb{N^*} \end{array} \right.$ Tính $\lim \left( {\frac{{{x_{n + 1}}}}{{{x_1}{x_2}...{x_n}}}} \right)$

Giải: Đặt ${x_n} = 2{u_n}$ ta được dãy số $({u_n})$ định bởi: $\left\{ \begin{array}{l} {u_n} = 2\\ {u_{n + 1}} = 2u_n^2 - 1\,,\,\,\forall n \in \mathbb{N^*} \end{array} \right.$

Đặt $ch\alpha = 2 \Leftrightarrow \frac{{{e^\alpha } + {e^{ - \,\alpha }}}}{2} = 2$, giải phương trình ta chọn nghiệm ${e^\alpha } = 2 + \sqrt 3 $

Chứng minh quy nạp ta được: ${u_n} = ch\left( {{2^{n - 1}}\alpha } \right)$ suy ra ${x_n} = 2ch\left( {{2^{n - 1}}\alpha } \right)$

Ta có: $${x_{n + 1}} = 2ch\left( {{2^n}\alpha } \right) = {\left( {{e^\alpha }} \right)^{{2^n}}} + {\left( {{e^{ - \,\alpha }}} \right)^{{2^n}}} = {\left( {2 + \sqrt 3 } \right)^{{2^n}}} + {\left( {2 - \sqrt 3 } \right)^{{2^n}}}$$
$${x_1}{x_2}...{x_n} = {2^n}ch\left( \alpha \right).ch\left( {2\alpha } \right)....ch\left( {{2^{n - 1}}\alpha } \right) = \frac{{{2^n}}}{{sh\alpha }}sh\left( \alpha \right).ch\left( \alpha \right).ch\left( {2\alpha } \right)....ch\left( {{2^{n - 1}}\alpha } \right)$$
\[ = \frac{{sh\left( {{2^n}\alpha } \right)}}{{sh\alpha }} = \frac{{{{\left( {{e^\alpha }} \right)}^{{2^n}}} - {{\left( {{e^{ - {\kern 1pt} \alpha }}} \right)}^{{2^n}}}}}{{{e^\alpha } - {e^{ - {\kern 1pt} \alpha }}}} = \frac{{{\rm{ }}{{\left( {2 + \sqrt 3 } \right)}^{{2^n}}} - {{\left( {2 - \sqrt 3 } \right)}^{{2^n}}}}}{{2\sqrt 3 }}\]
Từ đó dễ dàng tính được $\lim \left( {\frac{{{x_{n + 1}}}}{{{x_1}{x_2}...{x_n}}}} \right) = 2\sqrt 3 $

Ví dụ 10: Tìm số hạng tổng quát của dãy số $({u_n})$ định bởi: $\left\{ \begin{array}{l} {u_1} = 3\\ {u_{n + 1}} = \left( {8u_n^3 + 4{u_n}} \right)\sqrt {1 + u_n^2} \,,\,\,\forall n \in \mathbb{N^*} \end{array} \right.$

Giải: Đặt $sh\alpha = 3 \Leftrightarrow \frac{{{e^\alpha } - {e^{ - \,\alpha }}}}{2} = 3$. Giải phương trình ta được: ${e^\alpha } = 3 + \sqrt {10} $

Giả sử ${u_n} = sh\left( {{4^{n - 1}}\alpha } \right)$.

Khi đó: ${u_{n + 1}} = \left[ {8s{h^3}\left( {{4^{n - 1}}\alpha } \right) + 4sh\left( {{4^{n - 1}}\alpha } \right)} \right]\sqrt {1 + s{h^2}\left( {{4^{n - 1}}\alpha } \right)} = sh\left( {{4^n}\alpha } \right)$

Vậy: ${u_n} = sh\left( {{4^{n - 1}}\alpha } \right) = \frac{1}{2}\left[ {{{\left( {{e^\alpha }} \right)}^{{4^{n - 1}}}} - {{\left( {\frac{1}{{{e^\alpha }}}} \right)}^{{4^{n - 1}}}}} \right] = \frac{1}{2}\left[ {{{\left( {\sqrt {10} + 3} \right)}^{{4^{n - 1}}}} - {{\left( {\sqrt {10} - 3} \right)}^{{4^{n - 1}}}}} \right]$

Ví dụ 11: (Đề thi Olympic đề nghị của trường Lê Hồng Phong TPHCM năm 2009)

Tìm số hạng tổng quát của dãy số $({u_n})$ định bởi: $\left\{ \begin{array}{l} {u_1} = m\\ {u_{n + 1}} = u_n^4 - 12u_n^3 + 50u_n^2 - 84{u_n} + 50\,,\,\,\forall n \in \mathbb{N^*} \end{array} \right.$

Giải: Đặt ${u_n} = {x_n} + 3$ ta được: $\left\{ \begin{array}{l} {x_1} = m - 3\\ {x_{n + 1}} = x_n^4 - 4x_n^2 + 2\,,\,\,\forall n \in \mathbb{N^*} \end{array} \right.$

Đặt ${x_n} = 2{v_n}$ ta được: $\left\{ \begin{array}{l} {v_1} = \frac{{m - 3}}{2}\\ {v_{n + 1}} = 8v_n^4 - 8v_n^2 + 1\,,\,\,\forall n \in \mathbb{N^*} \end{array} \right.$

* Nếu $m = 5$ ta được ${v_n} = 1$ suy ra ${u_n} = 5,\,\, \forall n \in \mathbb{N^*}$

* Nếu $m = 1$ ta được ${u_1} = 1,\, {u_n} = 5, \,\forall n \ge 2$

* Nếu $\left| {\frac{{m - 3}}{2}} \right| < 1 \Leftrightarrow 1 < m < 5$. Đặt ${v_1} = \cos \alpha $

Áp dụng công thức $\cos 4\alpha = 8{\cos ^4}\alpha - 8{\cos ^2}\alpha + 1$ ta được ${v_2} = \cos \left( {4\alpha } \right)$

Chứng minh quy nạp ta được ${v_n} = \cos \left( {{4^{n - 1}}\alpha } \right)$ suy ra ${u_n} = 2\cos \left( {{4^{n - 1}}\alpha } \right) + 3\,,\,\,\forall n \in \mathbb{N^*}$

* Nếu $\frac{{m - 3}}{2} > 1 \Leftrightarrow m > 5$. Đặt $ch\alpha = \frac{{m - 3}}{2} \Leftrightarrow \frac{{{e^\alpha } + {e^{ - \,\alpha }}}}{2} = \frac{{m - 3}}{2} \Leftrightarrow {e^{2\alpha }} - \left( {m - 3} \right){e^\alpha } + 1 = 0$

Giải phương trình ta lấy nghiệm ${e^\alpha } = \frac{{m - 3 + \sqrt {{m^2} - 6m + 5} }}{2}$

Chứng minh quy nạp ta được: ${v_n} = \frac{1}{2}\left( {{{\left( {{e^\alpha }} \right)}^{{4^{n - 1}}}} + {{\left( {{e^{ - \,\alpha }}} \right)}^{{4^{n - 1}}}}} \right)$

Suy ra: ${u_n} = {\left( {{e^\alpha }} \right)^{{4^{n - 1}}}} + {\left( {{e^{ - \,\alpha }}} \right)^{{4^{n - 1}}}} + 3\,,\,\,\forall n \in \mathbb{N^*}$

* Nếu $\frac{{m - 3}}{2} < - 1 \Leftrightarrow m < 1$. Đặt $ch\alpha = \frac{{3 - m}}{2} \Leftrightarrow \frac{{{e^\alpha } + {e^{ - \,\alpha }}}}{2} = \frac{{3 - m}}{2} \Leftrightarrow {e^{2\alpha }} + \left( {m - 3} \right){e^\alpha } + 1 = 0$

Giải phương trình ta lấy nghiệm ${e^\alpha } = \frac{{3 - m + \sqrt {{m^2} - 6m + 5} }}{2}$

Chứng minh quy nạp ta được: ${v_n} = \frac{1}{2}\left( {{{\left( {{e^\alpha }} \right)}^{{4^{n - 1}}}} + {{\left( {{e^{ - \,\alpha }}} \right)}^{{4^{n - 1}}}}} \right)\,,\,\,\forall n \ge 2$

Suy ra: ${u_n} = {\left( {{e^\alpha }} \right)^{{4^{n - 1}}}} + {\left( {{e^{ - \,\alpha }}} \right)^{{4^{n - 1}}}} + 3\,,\,\,\forall n \ge 2$

Ví dụ 12: (Trích đề thi chọn đội tuyển TPHCM _2012)

Tìm số hạng tổng quát của dãy số $({x_n})$ định bởi: $\left\{ \begin{array}{l} {x_1} = \frac{4}{5}\\ {x_{n + 1}} = \frac{{x_n^4}}{{x_n^4 - 8x_n^2 + 8}}\,,\,\forall n \in \mathbb{N^*} \end{array} \right.$

Giải: Từ giả thiết ta được ${x_n} > 0,\,\,\forall n \in \mathbb{N^*}$ và $\frac{1}{{{x_{n + 1}}}} = 8{\left( {\frac{1}{{{x_n}}}} \right)^4} - 8{\left( {\frac{1}{{{x_n}}}} \right)^2} + 1$

Đặt ${u_n} = \frac{1}{{{x_n}}}$ ta được dãy số $({u_n})$ định bởi: $\left\{ \begin{array}{l} {u_1} = \frac{5}{4}\\ {u_{n + 1}} = 8u_n^4 - 8u_n^2 + 1,\,\,\forall n \in \mathbb{N^*} \end{array} \right.$

Đặt $ch\alpha = \frac{5}{2} \Leftrightarrow \frac{{{e^\alpha } + {e^{ - \,\alpha }}}}{2} = \frac{5}{4}$, giải phương trình ta chọn nghiệm ${e^\alpha } = 2$

Chứng minh quy nạp ta được: $${u_n} = ch\left( {{4^{n - 1}}\alpha } \right) = \frac{{{{\left( {{e^\alpha }} \right)}^{{4^{n - 1}}}} + {{\left( {{e^{ - \,\alpha }}} \right)}^{{4^{n - 1}}}}}}{2} = \frac{{{2^{^{{4^{n - 1}}}}} + {{\left( {\dfrac{1}{2}} \right)}^{{4^{n - 1}}}}}}{2}$$
(Áp dụng công thức: $ch(4x) = 8ch^4x – 8ch^2x + 1$)

Vậy: ${x_n} = \frac{2}{{{2^{{4^{n - 1}}}} + {{\left( {\dfrac{1}{2}} \right)}^{{4^{n - 1}}}}}}$

Ví dụ 13: Cho hai dãy số $({x_n})$ và $({y_n})$ xác định như sau: ${x_1}=a>0,\,{y_1}=b>0,\,\,\,{x_{n + 1}} = \frac{{{x_n} + {y_n}}}{2}$, ${y_{n + 1}} = \sqrt {{x_{n + 1}}.{y_n}} \,\,,\,\,\forall n \in \mathbb{N^*}$. Tìm $\lim {x_n}\,\,\text{và}\,\,\lim {y _n}$

Giải: Ta xét các trường hợp sau:

* Trường hợp 1: Nếu $a = b$ thì ${x_n} = {y_n} = a, \,\,\forall n \in \mathbb{N^*}$ nên $lim{x_n} = lim{y_n} = 1$

* Trường hợp 2: Nếu $a < b$ thì đặt $\cos \alpha = \frac{a}{b}\,\,\,,\,\,\alpha \in \left( {0\,,\,\frac{\pi }{2}} \right)$. Khi đó ta có:
$${x_2} = \frac{{b\left( {1 + \dfrac{a}{b}} \right)}}{2} = \frac{{b\left( {1 + \cos \alpha } \right)}}{2} = b{\cos ^2}\frac{\alpha }{2}\,\,\,\text{và}\,\,\, {y_2} = \sqrt {{x_2}{y_1}} = \sqrt {{b^2}{{\cos }^2}\frac{\alpha }{2}} = b\cos \frac{\alpha }{2}$$
$${x_3} = \frac{{{x_2} + {y_2}}}{2} = \dfrac{{b\cos \dfrac{\alpha }{2}\left( {1 + \cos \dfrac{\alpha }{2}} \right)}}{2} = b\cos \frac{\alpha }{2}\cos \frac{\alpha }{{{2^2}}}\,\,\,\text{và}\,\,\, {y_3} = \sqrt {{x_3}{y_2}} = b\cos \frac{\alpha }{2}\cos \frac{\alpha }{{{2^2}}}$$
Chứng minh quy nạp ta được:
$${x_n} = b\left( {\cos \frac{\alpha }{2}.\cos \frac{\alpha }{{{2^2}}}....\cos \frac{\alpha }{{{2^{n - 1}}}}} \right){\cos ^2}\frac{\alpha }{{{2^n}}}\,\,\,\text{và}\,\,\,{y_n} = b\cos \frac{\alpha }{2}\cos \frac{\alpha }{{{2^2}}}...\cos \frac{\alpha }{{{2^n}}},\,\forall n \ge 2$$
Áp dụng công thức: $\cos x = \frac{{{\mathop{\rm s}\nolimits} {\rm{in2}}x}}{{2\sin x}}$ ta rút gọn được ${y_n} = b\frac{{\sin \alpha }}{{{2^n}\sin \dfrac{\alpha }{{{2^n}}}}}$

Vì $\lim \left( {{2^n}\sin \frac{\alpha }{{{2^n}}}} \right) = \lim \left( {\alpha \frac{{\sin \dfrac{\alpha }{{{2^n}}}}}{{\dfrac{\alpha }{{{2^n}}}}}} \right) = \alpha $ nên $\lim {y_n} = \frac{{b\sin \alpha }}{\alpha }$

Từ ${x_n} = {y_n}\cos \frac{\alpha }{{{2^n}}} \Rightarrow \lim {x_n} = \lim \left( {{y_n}\cos \frac{\alpha }{{{2^n}}}} \right) = \left( {\lim {y_n}} \right)\left( {\lim \cos \frac{\alpha }{{{2^n}}}} \right) = \frac{{b\sin \alpha }}{\alpha }$

* Trường hợp 3: Nếu $a > b$, chọn số $\alpha$ sao cho ${\mathop{\rm ch}\nolimits} \alpha = \frac{a}{b}$

$${x_2} = \frac{{b\left( {1 + \dfrac{a}{b}} \right)}}{2} = \frac{{b\left( {1 + {\mathop{\rm ch}\nolimits} \alpha } \right)}}{2} = b{{\mathop{\rm ch}\nolimits} ^2}\frac{\alpha }{2}\,\,\,\text{và}\,\,\, {y_2} = \sqrt {{x_2}{y_1}} = \sqrt {{b^2}{{{\mathop{\rm ch}\nolimits} }^2}\frac{\alpha }{2}} = b{\mathop{\rm ch}\nolimits} \frac{\alpha }{2}$$
$${x_3} = \frac{{{x_2} + {y_2}}}{2} = \frac{{b{\mathop{\rm ch}\nolimits} \dfrac{\alpha }{2}\left( {1 + {\mathop{\rm ch}\nolimits} \dfrac{\alpha }{2}} \right)}}{2} = b{\mathop{\rm ch}\nolimits} \frac{\alpha }{2}{\mathop{\rm ch}\nolimits} \frac{\alpha }{{{2^2}}}\,\,\,\text{và}\,\,\, {y_3} = \sqrt {{x_3}{y_2}} = b{\mathop{\rm ch}\nolimits} \frac{\alpha }{2}{\mathop{\rm ch}\nolimits} \frac{\alpha }{{{2^2}}}$$
Chứng minh quy nạp ta được:
$${x_n} = b\left( {{\mathop{\rm ch}\nolimits} \frac{\alpha }{2}.{\mathop{\rm ch}\nolimits} \frac{\alpha }{{{2^2}}}....{\mathop{\rm ch}\nolimits} \frac{\alpha }{{{2^{n - 1}}}}} \right){{\mathop{\rm ch}\nolimits} ^2}\frac{\alpha }{{{2^n}}}\,\,\,\text{và}\,\,\,{y_n} = b{\mathop{\rm ch}\nolimits} \frac{\alpha }{2}{\mathop{\rm ch}\nolimits} \frac{\alpha }{{{2^2}}}...{\mathop{\rm ch}\nolimits} \frac{\alpha }{{{2^n}}},\,\,\forall n \ge 2$$
Áp dụng công thức: ${\mathop{\rm ch}\nolimits} x = \frac{{sh\,{\rm{2}}x}}{{2shx}}$ ta rút gọn được ${y_n} = b\frac{{{\mathop{\rm sh}\nolimits} \alpha }}{{{2^n}.{\mathop{\rm sh}\nolimits} \dfrac{\alpha }{{{2^n}}}}}$

Đặt $x = \frac{\alpha }{{{2^n}}}$, khi $n\, \to + \infty $ thì $x \to 0$
$$\lim \left( {{2^n}.{\mathop{\rm sh}\nolimits} \frac{\alpha }{{{2^n}}}} \right) = \mathop {\lim }\limits_{x \to 0} \frac{{\alpha \left( {{e^x} - {e^{ - x}}} \right)}}{{2x}} = \mathop {\lim }\limits_{x \to 0} \left( {\frac{\alpha }{{{e^x}}}.\frac{{{e^{2x}} - 1}}{{2x}}} \right) = \alpha$$
$$\text{nên}\,\,\,\lim {y_n} = \frac{{b{\mathop{\rm sh}\nolimits} \alpha }}{\alpha },\,\,\,\lim \left( {{\mathop{\rm ch}\nolimits} \frac{\alpha }{{{2^n}}}} \right) = \mathop {\lim }\limits_{x\, \to \,0} \frac{{{e^x} + {e^{ - x}}}}{2} = 1$$
$$\text{và}\,\,\,{x_n} = {y_n}.\cosh \frac{\alpha }{{{2^n}}} \Rightarrow \lim {x_n} = \lim \left( {{y_n}.{\mathop{\rm ch}\nolimits} \frac{\alpha }{{{2^n}}}} \right) = \left( {\lim {y_n}} \right)\left( {\lim {\mathop{\rm ch}\nolimits} \frac{\alpha }{{{2^n}}}} \right) = \frac{{b{\mathop{\rm sh}\nolimits} \alpha }}{\alpha }.$$

II. Bài tập áp dụng

Bài 1: Tìm công thức tính số hạng tổng quát của các dãy số sau:
a) $\left\{ \begin{array}{l} {u_1} = 1\\ {u_{n + 1}} = 5u_n^2 - \frac{2}{5}\,,\,\,\forall n \in \mathbb{N^*} \end{array} \right.$
b) $\left\{ \begin{array}{l}{u_1} = 1\\ {u_{n + 1}} = 5u_n^2 - \frac{2}{5}\,,\,\,\forall n \in \mathbb{N^*} \end{array} \right.$
c)
$\left\{ \begin{array}{l}u_1 = \frac{1}{3}\\u_{n + 1} = 6u_n^2 + 8u_n + \frac{5}{3}\,\,\,\forall n \in \mathbb{N^*}\end{array} \right.$
d) $\left\{ \begin{array}{l}{u_1} = 20\\{u_{n + 1}} = u_n^2 - {2.5.2^n}\,,\,\,\forall n \in \mathbb{N^*}\end{array} \right.$
e) $\left\{ \begin{array}{l}{u_1} = 5\\{u_{n + 1}} = {2.3^{{2^n}}}u_n^2 - {3^{\left( {n + 1} \right){2^n}}}\,,\,\,\forall n \in \mathbb{N^*}\end{array} \right.$
f) $\left\{ \begin{array}{l}{u_1} = 0\\{u_{n + 1}} = \frac{{\sqrt {3{u_n} + 8} }}{3} - 2\,,\,\,\forall n \in \mathbb{N^*}\end{array} \right.$

Bài 2: Tìm công thức tính số hạng tổng quát của các dãy số sau:
a) $\left\{ \begin{array}{l}{u_1} = - 3\\{u_{n + 1}} = 2{u_n}\sqrt {1 + u_n^2} \,,\,\,\forall n \in \mathbb{N^*}\end{array} \right.$
b) $\left\{\begin{array}{l}{u_1} = - \,2\\{u_{n + 1}} = \frac{1}{2}{u_n} + \frac{1}{{{u_n}}}\,,\,\,\forall n \in\mathbb{N^*}\end{array} \right.$
c) $\left\{ \begin{array}{l}{u_1} = 1\\{u_{n + 1}} = 2{u_n}\sqrt {u_n^2 - 4{u_n} + 5} - 4\sqrt {u_n^2 - 4{u_n} + 5} + 2\,,\,\,\forall n \in \mathbb{N^*}\end{array} \right.$
d)
$\left\{ \begin{array}{l}u_1 = 3\\u_{n + 1} = \dfrac{u_n^2 - 4}{u_n^2 + 2u_n + 2}\,\,\,\forall n \in \mathbb{N^*} \end{array} \right.$
e) $\left\{ \begin{array}{l}u_1 = 5\\u_{n + 1} = 3u_n + 2\sqrt {2u_n^2 - 2} \,\,\,\forall n \in \mathbb{N^*} \end{array} \right.$
f) $\left\{ \begin{array}{l}{u_1} = - \,2\\{u_{n + 1}} = 2\sqrt {u_n^2 + 1} + \sqrt 5 {u_n}\,,\,\,\forall n \in \mathbb{N^*}\end{array} \right.$

Bài 3: Tìm công thức tính số hạng tổng quát của các dãy số sau:
a)
$\left\{ \begin{array}{l}u_1 = \dfrac{{\sqrt {2 + \sqrt 2 } }}{2} - 1\\u_{n + 1} = 4u_n^3 + 12u_n^2 + 9u_n\,\,\,\forall n \in \mathbb{N^*}\end{array} \right.$
b) $\left\{ \begin{array}{l} {u_1} = \frac{3}{2}\\ {u_{n + 1}} = 4u_n^3 - 6u_n^2 + 6{u_n} - \frac{3}{2}\,,\,\,\forall n \in \mathbb{N^*} \end{array} \right.$
c) $\left\{ \begin{array}{l}u_1 = 1\\u_{n + 1} = \dfrac{u_n^3 - 3u_n - 6}{3u_n^2 + 6u_n +7}\,\,\,\forall n \in \mathbb{N^*}\end{array} \right.$

Bài 4: Tìm công thức tính số hạng tổng quát của các dãy số sau:
a)
$\left\{ \begin{array}{l} u_1 = 3\\ u_{n + 1} = 8u_n^4 - 64u_n^3 + 184u_n^2 - 224u_n + 99\,\,\,\forall n \in \mathbb{N^*} \end{array} \right.$
b) $\left\{ \begin{array}{l} {u_1} = 1\\ {u_{n + 1}} = \dfrac{{u_n^4}}{{u_n^4 + 8u_n^2 + 8}}\,\,\,\forall n \in \mathbb{N^*} \end{array} \right.$
c) $\left\{ \begin{array}{l} u_1 = 5\\ u_{n + 1} = \dfrac{u_n^4}{\left( 8 + 4u_n \right)\sqrt {1 + u_n^2}} \end{array} \right.$
d) $\left\{ \begin{array}{l} u_1 = 2\\ u_{n + 1} = \dfrac{1 + 3u_n^2}{u_n^3 + 3u_n}\,\,\,\forall n \in \mathbb{N^*} \end{array} \right.$
e) $\left\{ \begin{array}{l} u_1 = 1\\ u_{n + 1} = \frac{u_n^3 + 6u_n^2 + 15u_n + 14}{3u_n^2 + 12u_n + 13}\,\,\,\forall n \in \mathbb{N^*} \end{array} \right.$
Mời các bạn cùng thảo luận tại: http://diendantoanho...showtopic=73743